Download as pdf or txt
Download as pdf or txt
You are on page 1of 186

NCEA

LEVEL 2
MATHEMATICS
QUESTIONS AND ANSWERS

P J Kane

Published by Mahobe Resources (NZ) Ltd


The NZ Centre of Mathematics
mathscentre
Free instructional videos
Free maths workbooks
Free worksheets
Free practice exams
Free interactive maths

The NZ Centre of Mathematics, is the access point for quality


mathematical videos, textbooks and learning material on all the
important mathematical topics. Each topic can be found in the New
Zealand Mathematics Curriculum and NCEA examination program. All
of the mathematics on the site is free of charge to download or watch.

www.mathscentre.co.nz

Proudly sponsored by Mahobe Resources (NZ) Ltd

Your first access point for school textbooks and calculators.

www.mahobe.co.nz MAHOBE
ACKNOWLEDGEMENTS

NCEA Level 2 Mathematics


Questions & Answers
P. J. Kane

This eBook was published in 2010.

Mahobe Resources (NZ) Ltd


P.O. Box 109-760
Newmarket, Auckland
New Zealand

www.mahobe.co.nz

© Mahobe Resources (NZ) Ltd


ISBN 9781877216824

This eBook has been provided by Mahobe Resources (NZ) Ltd to The New Zealand Centre of Mathematics.
Schoolteachers, University lecturers, and their students are able to freely download this book from The New
Zealand Centre of Mathematics website www.mathscentre.co.nz. Electronic copies of the complete eBook may
not be copied or distributed. Students have permission to print one copy for their personal use. Any photocopying
by teachers must be for training or educational purposes and must be recorded and carried out in accordance with
Copyright Licensing Ltd guidelines. The content presented within the book represents the views of the publisher
and his contributors as at the date of publication. Because of the rate with which conditions change, the publisher
and his contributors reserve the right to alter and update the contents of the book at any time based on the new
conditions. This eBook is for informational purposes only and the publisher and his contributors do not accept any
responsibilities for any liabilities resulting from the use of the information within. While every attempt has been
made to verify the content provided, neither the publisher nor his contributors and partners assume any
responsibility for errors, inaccuracies oromissions. All rights reserved. All the views expressed in this book are
those of the author. The questions and suggested answers are the responsibility of the author and have not been
moderated for use in NCEA examinations.

Thats all the legal stuff over. We hope that the book is helpful!
3

CONTENTS

2.1 Algebra . . . . . . . . . . . . . . . . . . . . . . . . . . . . 5

2.2 Graphs . . . . . . . . . . . . . . . . . . . . . . . . . . . . 19

2.3 Calculus . . . . . . . . . . . . . . . . . . . . . . . . . . . 39

2.4 Co-ordinate Geometry . . . . . . . . . . . . . . . . . . 55

2.5 Sample Statistics . . . . . . . . . . . . . . . . . . . . . 73

2.6 Probability & Normal Distribution . . . . . . . . . . 87

2.7 Sequences . . . . . . . . . . . . . . . . . . . . . . . . 105

2.8 Trigonometry Problems . . . . . . . . . . . . . . . . 117

2.9 Trigonometric Equations . . . . . . . . . . . . . . . 130

The Answers . . . . . . . . . . . . . . . . . . . . . . . 144

Areas Under the Normal Curve . . . . . . . . . . . 171

Formulae Sheet . . . . . . . . . . . . . . . . . . . . . 172

Pages for Extra Notes . . . . . . . . . . . . . . . . . 173

YEAR 12 MATHEMATICS
4

STUDYING NCEA LEVEL 2 MATHS

Ø This book has been written for you to practise NCEA Level Two- type assessments. Nine chapters have
been designed to match the nine achievement standards at this level.

Ù Each chapter begins with a schedule of the requirements for that achievement standard. As you read
down each schedule, you will see that the challenges become more complex.

Ú In most chapters a preliminary set of exercises has been provided to set in motion the set of skills
required for the achievement objectives. Once you think that you have mastered the skill set progress
onto the first model assessment.

For external achievement standards allow 45 - 60 minutes.

For internal achievement standards allow 4-5 hours as these are more project orientated.

Check the solutions, and if yours do not quite match these, rework your calculations, or check with
friends or teachers until you are satisfied.

Û Attempt the second model assessment 1-2 weeks later to see if the themes you covered still ‘click’.
Again, check the solutions at the back of the book with yours. Remember, you can still learn from your
mistakes .... this side of the final exams.

Ü It is worth recognising that in Year 12 your mathematics may appear to have begun at a roundabout.
It introduces new themes which seem to go down different roads. One of the strengths of this subject,
however, is that these themes or roads are connected, though this may not be evident just yet.

Therefore as you are being assessed in discrete themes or standards, try to develop an eye for the
bigger picture. As always mathematics is about solving problems and finding patterns and reasons.
Hopefully your experiences this year will provide you with confidence and judgement for future
challenges.

YEAR 12 MATHEMATICS
The DS-742ET

Did you know that Mahobe added equation solving to


make this an even more powerful calculator?

eTOOL

-100 -50 0 50 100 MAHOBE

www.mahobe.co.nz.
Algebra
5

MATHEMATICS 2.1
ACHIEVEMENT STANDARD 90284

Manipulate algebraic expressions and solve equations

Below are the requirements of this Achievement Standard.

C expand brackets (up to 3 bracket sets)


C factorise expressions including quadratics
C simplify and use fractional exponents
From the
C simplify and use integer exponents
C interchange between exponential and straightforward
logarithmic statements
(less steps)
C solve linear equations or inequations using at
cases
least 2 steps
C solve quadratic equations which can be
factorised to
C solve simple logarithmic equations
C form then solve pairs of linear simultaneous
situations
equations
involving
C solve quadratics using the quadratic formula
C solve a pair of simultaneous equations, with more depth,
one being linear and one being non-linear
more steps
C solve exponential equations, which may involve
logarithmic methods and

C complete algebraic challenges such as proving growing complexities


an algebraic statement with
sensible interpretations
C explore the nature of the roots of a quadratic
of the solutions(s).
equation.

YEAR 12 MATHEMATICS
Algebra
6

ALGEBRA - Revision Summary


The exercises in this section have been specifically chosen to underpin the Achievement Criteria that will be
tested in this Achievement Standard. Write your answers on the opposite page.

1. ASSUMED KNOWLEDGE
Rearrange (change the subject of) these formulae:
a. x, in y = 3x - 7
b. r, in A = 4πr2

Solve in factored form:


c. (2x - 5)(x + 11) = 0
d. 5x2(1 - 3x)2(x + 8) = 0

2. EXPAND BRACKETS AND SIMPLIFY


a. 3x(2x - 3) - 6(x2 - 2)
b. 2x(3x + 1)(3x - 5)
c. (x + 3)(2x - 7)2

3. FACTORISE ALGEBRAIC EXPRESSIONS


Factorise completely:
a. x2 - 19x + 84
b. 10x2 - x - 21
c. a2 - av + aw - vw

4. FRACTIONAL AND NEGATIVE INDICES

a. Write with positive indices.

b. Simplify .

c. Simplify .

5. ELEMENTARY PROPERTIES OF LOGARITHMS


a. Write log3 243 = 5 in exponential form.
b. Express as the log of a single number:
i. log 15 - log 3
ii. 3 log 2 + 2 log 3

iii. 2 log 6 - log 36 + log 5

c. Simplify

YEAR 12 MATHEMATICS
Algebra
7

YEAR 12 MATHEMATICS
Algebra
8

6. SIMPLIFY RATIONAL EXPRESSIONS

a.

b.

c.

d. ×

7. SOLVE LINEAR EQUATIONS AND INEQUATIONS

a. x-3=x+7

b. 2(n + 3) = 5(n - 1) - 7(2n - 3)


c. 3(2x + 1) < 2x + 9

d. 2(x + 3) >

8. SOLVE QUADRATIC EQUATIONS


By factorising
a. x2 - x - 42 = 0
b. 3x2 - 7x + 2 = 0
c. 5x = 2x3 + 3x2

By quadratic formula (to 2dp)


d. 4x2 - 2x - 3 = 0
e. x2 + 4x - 2 = 0
f. 8 - x - x2 = 0

9. SOLVE LOGARITHMIC AND EXPONENTIAL EQUATIONS


Find the value of x in:
a. log3 x = 7
b. logx 343 = 3
c. 7x-1 = 26

YEAR 12 MATHEMATICS
Algebra
9

YEAR 12 MATHEMATICS
Algebra
10

10. SOLVE SIMULTANEOUS EQUATIONS


Linear pairs
a. x + 3y = 5 b. y=x+5
2x + y = -5 3y + 4x = 1

Line and curve


c. y = x2 - 3x d. xy = 2
y = 2x - 6 y-x=1

11. QUADRATIC ROOTS


a. Find the nature of the roots of:
i. 4x2 - 13x + 7 = 0
ii. 25x2 - 30x + 9 = 0
b. Use the discriminant of 2x2 - 2nx + 5 = 0 to find the values of n for which
there will be no real roots (i.e. imaginary).

YEAR 12 MATHEMATICS
Algebra
11

ALGEBRA - PRACTICE TEST 1


QUESTION ONE

1. Simplify:

2. Simplify fully:

3. Write as the log of a single number: log 112 - log 14

4. Solve the following equations:

a. =

b. logx 44 = 5

c. 3x2 - x = 4

YEAR 12 MATHEMATICS
Algebra
12

QUESTION TWO

A suburb in a major city has been infected by a foreign moth which could have devastating effects on
neighbouring farms and forests. An aerial spray campaign is launched where an aeroplane flies over the area
and spreads an insecticide, which though fatal to the moth is harmless to humans and other creatures.

The formula M = M0(0.85)t gives the number of moths (M) in the spray zone t days after the plane has sprayed.
M0 is the initial number of moths that the Ministry officials believe were in the zone. If they believe that 800
moths were present in the zone, how many days after spraying would it take the population to fall to 500
moths?

QUESTION THREE

A circular traffic island in the middle of an intersection is planned. The circle is represented by x2 + y2 = 36.
Also in the plan is a path of an electrical cable which runs underneath the traffic island. The cable path may
be shown by y = 2x + 6.
a. Find the x ordinates of the points where the cable meets the perimeter of the traffic island.
b. Find the y ordinates and hence write the points of intersection.

YEAR 12 MATHEMATICS
Algebra
13

QUESTION FOUR

A team of netballers and their supporters are fundraising in order to attend a Golden Oldies tournament in the
Cook Islands. One of their activities is a social at a local hall. The team has two options for pricing tickets to
this event.

Option Price A Option Price B

PA = PB =

Where x is the number of tickets sold Where x is the number of tickets sold
and the 9 best ticket sellers get free and the 12 best ticket sellers get free
tickets to the social. tickets to the social.

Solve PA = PB and find the minimum number of tickets which need to be sold so that the price of Option B
tickets would be less than the price of Option A tickets.

YEAR 12 MATHEMATICS
Algebra
14

QUESTION FIVE

The quadratic equation x2 - (k + 1)x + 4k = 0 has 2 roots.


If the difference between the roots is 1, find the value of k.

YEAR 12 MATHEMATICS
Algebra
15

ALGEBRA - PRACTICE TEST 2


QUESTION ONE

1. Expand and simplify: (x + 8)(3x - 1)(4x + 3)

2. Write this expression in positive index form:

3. Write as a single number:

4. Solve the equations:


a. logx 243 = 5 b. 3x2 - 8x = -4 c. 6x = 31

5. At the movies during the weekend, Moira served ice creams to a group of children from a birthday party.
Of the 9 she served, 7 wanted chocolate dipped while the other 2 wanted plain. If it cost a total of
$19.25 with a chocolate dipped ice cream being 50 cents more than a plain one, calculate the cost of
a plain ice cream.

YEAR 12 MATHEMATICS
Algebra
16

QUESTION TWO

Show that there is only one point of intersection between: x2 + y2 + 2x - 7 = 0 and y = x - 3.

QUESTION THREE

The height of a door is 1 metre longer than its width. The area of the door is 1.7 m2.
What are the dimensions of the door? (Give your answer to 1 dp.)

YEAR 12 MATHEMATICS
Algebra
17

QUESTION FIVE

After t hours of use, the value (V) of a certain brand of jetski (which was purchased new for $19 995) may be
estimated by: V = P(0.993)t where P is the retail price.

After how many hours of use would the jetski be worth of its original retail price?

YEAR 12 MATHEMATICS
Algebra
18

QUESTION SIX

A certain aeroplane can cover a distance of 5000 km travelling over a time, t hours, at a velocity v = .

If the same aeroplane flew the 5000 km again, this time increasing its speed by 250 km/h (i.e. v + 250), and
cutting the travelling time by an hour (i.e. t - 1), what would its speed have been in both instances?

YEAR 12 MATHEMATICS
The DS-742ET

Did you know that Mahobe added equation solving to


make this an even more powerful calculator?

eTOOL

-100 -50 0 50 100 MAHOBE

www.mahobe.co.nz.
Graphs
19

MATHEMATICS 2.2
ACHIEVEMENT STANDARD 90285

Draw straightforward non-linear graphs

Below are the requirements of this Achievement Standard.

C draw a quadratic graph whose equation


may be factorised, y = (cx - d)(x + e)
From more
or expressed as y = ±(x - a)2 + b
C draw a polynomial graph whose equation could
straightforward
be factorised (leading term (±1)xn )
C draw a rectangular hyperbola from an equation relations
such as y = or
and their
C draw a circle from a given equation whose
centre is at (0, 0) graphs
C draw an exponential function from a given
equation, y = ax to
C draw a logarithmic function from a given
equation y = loga x
graphs

C show relevant features including: intercepts, with more


maxima or minima, asymptotes, symmetry
complexity
C draw rectangular hyperbola y =
and more features.
2 2 2
C draw circles such as (x - a) + (y - b) = r
C draw exponential functions y = ax - b + c
Relations
C draw log functions y = loga(x - b) + c
given may have
C write equations and interpret features of any
coefficients
graphs of the above constraints and exponents
C model any of the above (or combinations of) to other than ± 1.
describe a situation, find points of intersection
and to solve related problems

YEAR 12 MATHEMATICS
Graphs
20

GRAPHS - Revision Summary


The exercises in this section have been specifically chosen to underpin the Achievement Criteria that will be
tested in this Achievement Standard.

1. NON LINEAR GRAPHS


Draw graphs for each of the following. Insure that you include any intercepts, asymptotes, symmetry
and maxima or minima for quadratic curves.
a. i. y = x2 + 3x - 4
ii. y = (x + 3)2 - 4
iii. (x + 1)(x - 2)(x + 3)

b. i. y= ii. y =

Draw each of the following pairs of graphs on the same axes.


c. i. x2 + y2 = 49 ii. (x + 2)2 + (y - 1)2 = 49
d. i. y = 5x ii. y = 5x - 3
e. i. y = log8 x ii. y = log8 *x - 3*

YEAR 12 MATHEMATICS
Graphs
21

YEAR 12 MATHEMATICS
Graphs
22

YEAR 12 MATHEMATICS
Graphs
23

2. For each of the graphs sketched below, write the equations.


a. b.

c. d.

YEAR 12 MATHEMATICS
Graphs
24

3. Draw the curve xy = -1 and y = x - 2 on the same axes.


Label any points of intersection with the correct co-ordinates.

YEAR 12 MATHEMATICS
Graphs
25

GRAPHS - PRACTICE TEST 1


QUESTION ONE

Draw the graphs of the three equations below.

a. y = x2 - 4x -5 b. y = c. y = 4x

YEAR 12 MATHEMATICS
Graphs
26

QUESTION TWO

1. Identify THREE features of the graphs y = x2 - 3x + 5 and y + x = 4.

2. Millie bought a car for $11995 some years ago. She knows that the current value of her car may be
modelled by the equation: V = $11995(0.82)t, where V is the current value of her car and t is the number
of years since she bought her car.
a. Plot the graph of this equation of the car’s current value over the six years since she bought it.
b. During which year did the value of the car fall under half of the purchase price?

1.

2.
V (Value in $)

t(years)

YEAR 12 MATHEMATICS
Graphs
27

QUESTION THREE

Draw graphs of these equations:


a. (x - 2)2 + (y + 3)2 = 9

b. y= = 3+

YEAR 12 MATHEMATICS
Graphs
28

QUESTION FOUR

Another car depreciation model that Millie discovered is given as: V = .

V is the value of the car (V) in dollars over t years.


The graph of the equation for the current value of the car is shown below.
a. What does the graph tell us about the rate at which the value of the car decreased?
b. What does the graph indicate about the value of the car after many years?
c. What does the y intercept tell us about the purchase price of the car?
V (Value in $)

t(years)

YEAR 12 MATHEMATICS
Graphs
29

QUESTION FIVE

For each of the graphs, write the equation.

y y
a. b.

x x

c.

YEAR 12 MATHEMATICS
Graphs
30

QUESTION SIX

Millie’s geology class has been studying volcanic crater lakes of the central North Island. One crater lake that
she studied had suddenly filled then burst one of its walls sending a torrent of water, mud and rock down the
mountain side. The data from the seismic monitoring station at the lake gave these figures:
Time Lake Depth Number of hours later
4pm, 10 Feb 6.6m (initial) 0
7am, 11 Feb 15.2m (burst) 15
10am, 13 Feb 8.0m 66

Millie models this situation with two hyperbolae (see graph below).

After the first 15 hours, the depth of the lake could be modelled by this hyperbole:

D is the depth of the crater lake (in metres) and t is the time (in hours) since the lake began to fill.
a. Write the equation for the (second) hyperbola which models the lakes depth after 15 hours.
b. Use your model equation above to estimate the time and the date when the crater lake returns to its
initial depth of 6.6 metres.

YEAR 12 MATHEMATICS
Graphs
31

GRAPHS - PRACTICE TEST 2


QUESTION ONE

a. Draw the graph of y = x(x - 1)(x + 3), showing all intercepts.


b. Draw the graph of y = 4 - (x + 1)2 showing key features.

YEAR 12 MATHEMATICS
Graphs
32

c. Draw the graph of y = log10 x.


d. Write three features of the circular graph illustrated below.

YEAR 12 MATHEMATICS
Graphs
33

QUESTION TWO

Write the equation of each of the following graphs.

a. b.

c.

YEAR 12 MATHEMATICS
Graphs
34

QUESTION THREE

Draw graphs of EACH of the following:


a. y = 2x2 - 3x - 5
b. y = -x3 + 1

YEAR 12 MATHEMATICS
Graphs
35

QUESTION FOUR

Draw the graph of y = 4 on the axes below for -5 # x #4.

YEAR 12 MATHEMATICS
Graphs
36

QUESTION FIVE

Helen and Don invest a sum of money into an education fund which compounds at 8% annually. The amount
in the account after t years may be given by the equation y = 45(1.08)t, where y the amount of money is in
hundreds of dollars. Below, a graph is given for the first 11 years.

a. What sum did Helen and Don invest initially?


b. If interest is calculated and added on at the end of every year, during which year would you expect
their original sum to have doubled?
c. 12 years after the fund began, Helen and Don need to withdraw $3000 for a family emergency. How
would this be represented on the graph?

YEAR 12 MATHEMATICS
Graphs
37

YEAR 12 MATHEMATICS
Graphs
38

QUESTION SIX

In the year of a general election a certain government department has been ordered to trim its spending (S)
according to this model equation: S = A - B log10 (x + 0.5), where S is in dollars, and x is the number of weeks
since the order was given.

By the end of Week 1, the Department has spent $107 449 for that week, but by the end of the tenth week,
their weekly spending was $64 011.
a. Find A and B (to the nearest $10), then rewrite the model equation with these values.
b. If the election was held seven months (30 weeks) after the order was given to the government
department, use your model equation to estimate how much had been spent by them in that election
week.

YEAR 12 MATHEMATICS
The DS-742ET

Mahobe have added some amazing technology into


their new eTool advanced scientific calculator.
• Equation solving.
• Enhanced statistics.
• Improved powers and fraction display.
This calculator is designed to handle even the toughest
assignments. If you use any other calculator then good
luck. With a Mahobe Resource you can have an added
confidence that the answer will be correct.

eTOOL

MAHOBE

www.mahobe.co.nz.
Calculus
39

MATHEMATICS 2.3
ACHIEVEMENT STANDARD 90286

Find and use straightforward derivatives and integrals

Below are the requirements of this Achievement Standard.

C find derivatives of polynomial expressions


such as 5x9 - 7x3 + 4
From more
C find integrals of polynomial expressions
C relate the derived function to the gradient
straightforward
of a curve
C relate the integral to the area under a curve uses of
C use the derivative to find the gradient at a point
and locate the point given a gradient value calculus techniques
C use the integral to find a straight forward
area under a curve, and to extract an equation and applications
given the gradient function
and familiarity with
C use differentiation techniques to locate turning , f!(x)
points where f!(x) = 0, then determine their
and Idx notations
nature(s), find the equation of a tangent to
a curve and solve rate of change problems
such as kinematics to

C using integration techniques to find areas wider ranging


(including compound) under polynomials applications and contexts
involving those
C use various calculus techniques to form techniques requiring
equations, to interpret results, to optimise interpretation of the
situations, to solve rates of change cases solutions.
(including kinematics) and to find relevant
areas

YEAR 12 MATHEMATICS
Calculus
40

CALCULUS - Revision Summary


The exercises in this section have been specifically chosen to underpin the Achievement Criteria that will be
tested in this Achievement Standard.

1. Differentiate these functions with respect to x:


a. y = 3x9 - 5x2 + 7
b. f(x) = (3x - 5)(2x2 + 7) (Hint: expand, then find f !(x).)

2. Find the indefinite integrals for:


a. f ! (x) = 3x2 + 8x - 11

b. = 18x5 + +1

3. a. Find f!(2) if f(x) = x4 - 5x2 + x.


b. Find the gradient of (the tangent to) the curve, y = x2 - 2x, at x = -1.

YEAR 12 MATHEMATICS
Calculus
41

4. For the curve y = x2 + 5x:


a. Find the equation of the tangent to this curve at (1, 6).
b. Find the equation of the tangent to this curve at x = 0.

5. Find the point on the curve y = x2 - 3x + 2 where the gradient is 1.

6. a. Evaluate the definite integral 3x2 . dx.

b. Find the area between the curve, y = 2x - x2 and the x axis between x = 0 and x = 2.
c. Find the total area between y = x(x - 1)(x + 3) and the x axis.

YEAR 12 MATHEMATICS
Calculus
42

7. Consider the curve, y = x3 - 3x = x(x2 - 3).

a. Find an expression for the gradient function .

b. Determine any turning points on y = x3 - 3x.


c. Along which values of x is the curve increasing and decreasing?

YEAR 12 MATHEMATICS
Calculus
43

8. A large model rocket is fired vertically into the air with an initial velocity of 245 m/s. After t seconds
the height of the rocket (h metres) is given by: h = 245t - 4.9t2.
a. Find an expression for the instantaneous velocity, v, of the rocket after t seconds.
b. What is the velocity of the rocket after 5 seconds?
c. What is the height of the rocket at the same time?
d. Show that the acceleration of the rocket is constant.
e. When does the rocket reach its maximum height above the ground, and what is this height?

YEAR 12 MATHEMATICS
Calculus
44

9. Optimisation situations require the use of calculus to find the maximum or minimum solution.
For example, in a new subdivision the developers wish to create rectangular sections, each having a
total boundary (or perimeter) of 108 m. What are the dimensions of such a rectangle, so that its area
could be a maximum?

YEAR 12 MATHEMATICS
Calculus
45

CALCULUS - PRACTICE TEST 1


Show ALL working.

QUESTION ONE

Find the gradient of the curve y = x3 - 6x - 5 at the point where x = 5.

QUESTION TWO

The graph shown below has the equation y = 3x2 + 1.


Calculate the shaded area.

YEAR 12 MATHEMATICS
Calculus
46

QUESTION THREE

The gradient function of a curve is f!(x) = 6x2 - 4x + 5.


The curve passes through the point (2, 11).
Find the equation of the curve.

QUESTION FOUR

Find the x co-ordinates of the two points on the graph of y = 2x3 - 6x + 8 where the gradient is parallel to the
x-axis.

YEAR 12 MATHEMATICS
Calculus
47

QUESTION FIVE

Find the equation of the tangent to the curve y = x3 - 3x2 - 7x + 1 at the point (-1, 4).

QUESTION SIX

Graeme returns to his car at the end of work and realises that he left the lights on, draining the battery.
Fortunately he parked on a slight slope earlier in the day, so he can roll the manual geared vehicle to push
start it.

As the vehicle slowly rolls forward, its velocity is given by v = 0.75t (m/s) where v = velocity in metres per
second and t = time in seconds from when the car begins to roll.

How far has the car rolled over the first 8 seconds?

YEAR 12 MATHEMATICS
Calculus
48

QUESTION SEVEN

Graeme designs rest areas along the edges of major highways. One of his more recent designs was the
computer designed area (part of which is shown as the shaded region on the graph below). As edges for the
area, he used these three equations:
y = 12 - 3x2
y = -36
and x = 1, where x and y are in metres.

Calculate the shaded (rest area) region .

YEAR 12 MATHEMATICS
Calculus
49

QUESTION EIGHT

To prevent flooding near a rural school the local council asked Graeme to design a drain along the rear
boundary. Graeme saves costs by designing a concrete structure to fit into an existing ditch.

Together the floor (width) and the heights of the two walls have a total
length of 5.6m.

Find the width of the floor which will allow the greatest flow of
stormwater through the cross-sectional area shown. Also, give this
maximum cross-sectional area.

YEAR 12 MATHEMATICS
Calculus
50

CALCULUS - PRACTICE TEST 2


Show ALL working.

QUESTION ONE

a. Find the gradient of the curve y = x4 - 3x2 + 5 at the point where x = 2.

b. Find the equation of the function which passes through the point (-1, 1) and whose gradient function

is = 8x3 + 6x2 - 4x - 1.

YEAR 12 MATHEMATICS
Calculus
51

c. Find the area under the curve, y = x3 + 2 for the values of x between 0 and 2.

d. Find the co ordinates of the point on the curve y = where the gradient is .

YEAR 12 MATHEMATICS
Calculus
52

QUESTION TWO

Find the area between the x - axis and the


curve y = (x + 1)(x - 4)
= x2 - 3x - 4

for values of x between 0 and 5.

YEAR 12 MATHEMATICS
Calculus
53

QUESTION THREE

3. An electronic powered model boat is being sailed on a small lagoon. Its velocity, in cm/s is given by:

v = 18 + 15t - 3t2 for 0 # t # 6


where t is the time in seconds after the boat is started.

a. After 2 seconds the boat is 65 cm from its owner who is controlling it from shore. How far was the
boat from the owner at the start?

b. Use calculus to find the maximum velocity of the boat.

YEAR 12 MATHEMATICS
Calculus
54

QUESTION FOUR

A manufacturer produces car polish in tin cans which have a volume of 335 cm3. Find the radius of the tin can
which requires the least amount of metal.

Note - for a cylinder V = πr2h and SA = 2πr2 + 2πrh.

YEAR 12 MATHEMATICS
The DS-742ET

Some advanced technology


has gone into the Mahobe
DS-742ET to make it one
of the most powerful
calculators available.

If you use anything else


then good luck!

planned orbit

eTOOL
actual orbit

C1 C2
www.mahobe.co.nz. MAHOBE
Coordinate Geometry
55

MATHEMATICS 2.4
ACHIEVEMENT STANDARD 90287

Use coordinate geometry methods

Below are the requirements of this Achievement Standard.

C find the mid-point between 2 points


C find the distance between 2 points From
C find the equation of a line
straightforward contexts
C find the equation of a parallel line in two dimensions
C find the equation of a perpendicular line
C find the coordinates of the point of intersection to
of 2 lines

situations with
C find equations of medians, perpendicular more complexity

bisectors and altitudes


including

C formulate a proof (i.e. geometric cases relying


on the above techniques three dimensional
possibilities

C prove points are collinear


and

C prove more challenging situations


extended chains of
reasoning.
C solve more challenging contextual problems

YEAR 12 MATHEMATICS
Coordinate Geometry
56

COORDINATE GEOMETRY - Revision Summary


The exercises in this section have been specifically chosen to underpin the Achievement Criteria that will be
tested in this Achievement Standard.

1. Find the midpoint between:


a. (0, 5) and (4, 11)
b. (-1.2, 6) and (4.4, -7.6)

2. Find the distances between the points in Question 1.

3. What is the gradient of the line 2y - 3x = 7 ?

YEAR 12 MATHEMATICS
Coordinate Geometry
57

4. Find the equation of the line which passes through (-3, 2) and (1, 5).

5. A line has the equation 2x + 5y - 8 = 0.


Write the equation of a line (in the form ax + by + c = 0) which is:
a. Parallel to and passing through (1, -2).
b. Perpendicular to and passing through (5, 6).

YEAR 12 MATHEMATICS
Coordinate Geometry
58

6. Find the points of intersection of these pairs of lines:


a. 3x + 4y = 10
5x + 3y = 13
b. 0.1x - 0.2y - 0.7 = 0
0.4x + 0.3y - 0.6 = 0

YEAR 12 MATHEMATICS
Coordinate Geometry
59

7. P(1, 5), Q(3, 2) and R(-3, -1) are the vertices of a triangle.
a. Find the equation of the median drawn from R to the midpoint of .
b. Find the equation of the altitude from P to .
c. Find the equation of the perpendicular bisector of .

YEAR 12 MATHEMATICS
Coordinate Geometry
60

8. Prove that the triangle ªOPQ is isosceles.

YEAR 12 MATHEMATICS
Coordinate Geometry
61

COORDINATE GEOMETRY - PRACTICE TEST 1


A training track for horses has the shape shown. Use coordinate geometry techniques to solve all questions.
Use the axes on the grid below to help answer these questions.
Note the grid lines are 20 m apart.

YEAR 12 MATHEMATICS
Coordinate Geometry
62

QUESTION ONE

a. Calculate the distance along the length of the back straight between N(4, 11) and S(-10, -2).

b. Find the equation of the line along this back straight.

c. A fence line passes through the point (-2, 8) and follows a path parallel to the line y = x + 5.

Find the equation of this fence line.

YEAR 12 MATHEMATICS
Coordinate Geometry
63

QUESTION TWO

A drain runs along a straight line equidistant between the points (1, -2) and (3, -4).
Find the equation of the line which the drain follows.

YEAR 12 MATHEMATICS
Coordinate Geometry
64

QUESTION THREE

The ends of the back straight N(4, 11) and S(-10, -2) form a triangle with a trough at T(2, 0).
Find the equation of the median of this triangle through N(4, 11).

YEAR 12 MATHEMATICS
Coordinate Geometry
65

QUESTION FOUR

The equation of the road between the ends S(-10, -2) on the back straight, and P(-2, -10) on the front straight
is x + y + 12 = 0.

The altitude of the triangle SPT, through the horse trough (2, 0) is given by the equation: x - y - 2 = 0.

Calculate the length of the altitude of the triangle SPT through vertex T(2, 0).

YEAR 12 MATHEMATICS
Coordinate Geometry
66

QUESTION FIVE

The farmer who owns the property wishes to move the back straight of the track so that it now runs along the

line y = x + 8.

Calculate the closest distance this new piece of track comes to a new trough planned at the position (6, 0).

YEAR 12 MATHEMATICS
Coordinate Geometry
67

COORDINATE GEOMETRY - PRACTICE TEST 2


Part of a mini golf course is shown, with the first five tees (T, symbol !) and holes (H, symbol F). A plan is
on the office wall, set to a grid system. The office has co-ordinates (0, 0) and some of the other tees and holes
have been given coordinates. Two electrical cables run under the pond, from T3 to T2 and from L3 to L4 and
are shown by dashed lines.

Every unit represents one metre.


The diagram is not drawn to scale.

YEAR 12 MATHEMATICS
Coordinate Geometry
68

QUESTION ONE

a. A solar light, L2, is located halfway between the second tee T2 (25, 15) and the second
hole H2 (11, 28). Find the coordinates of L2.

b. What is the equation of the line from T2 (25, 15) to T3 (1, 24)?

c. The equation of the path from hole two at H2 to the third tee, T3 is y = x- .

Write the equation of a line which is parallel to at and which passes through the bend

point, B(10, 26).

YEAR 12 MATHEMATICS
Coordinate Geometry
69

QUESTION TWO

An old cable running from the bend at C(19, 12), under the pond, meets the service path at the midpoint
between the two tees, T1(11, -2) and T3(1, 24). What is the length of this underground cable?

YEAR 12 MATHEMATICS
Coordinate Geometry
70

QUESTION THREE

Consider a triangle formed by the three points T3, H2 and T2.

Show how the equation of the altitude of the triangle T3, H2, T2 which passes through the vertex at T2 is
5x + 2y - 155 = 0.

YEAR 12 MATHEMATICS
Coordinate Geometry
71

QUESTION FOUR

The path for the fifth hole has two ‘legs’. The first leg starts at T5(9, 0) and runs perpendicular to the line T1
H1 until it gets to the bend at D. From D, the second leg runs along a path which is perpendicular to the line
BH3, ending at the hole H5 (3, 15).

What are the co-ordinates of the bend at D?

YEAR 12 MATHEMATICS
Coordinate Geometry
72

QUESTION FIVE

Another light is going to be located halfway between T3 and B so that the area of the north side of the
pond can be lit up. A cable will run from this point and be connected to the existing cable which runs
between T3 and T2.

What will be the shortest distance between the new light and the line T3 T2?

YEAR 12 MATHEMATICS
The DS-742ET

Mahobe have added some amazing technology into


their new eTool advanced scientific calculator.
• Equation solving.
• Enhanced statistics.
• Improved powers and fraction display.
This calculator is designed to handle even the toughest
assignments. If you use any other calculator then good
luck. With a Mahobe Resource you can have an added
confidence that the answer will be correct.

eTOOL

MAHOBE

www.mahobe.co.nz.
Sample Statistics
73

MATHEMATICS 2.5
ACHIEVEMENT STANDARD 90288

Select a sample and use this to make an inference about the population

Below are the requirements of this Achievement Standard.

C select a sample from a population


(possibly supplied) From
C provide evidence of the method
more straightforward
C use appropriate sample statistics such as: mean
inferences
median, quartiles, standard deviation, and taken from a sample
proportions from the selected sample about the population

C describe the sampling method so that another to


person could repeat the process
fuller justification
C comment on whether the sample is truly
of the method(s) used
representative, or shows bias
C refer to sample statistics (and suitable graphs) leading to

to help justify the above position a more critical


evaluation
of the whole sampling
C when evaluating the sampling process, consider
process
limitations of, and possible improvements
to reliability and

the results.
C evaluate the accuracy of the results, considering
an improved interval for the question

C refer to how the data is distributed

YEAR 12 MATHEMATICS
Sample Statistics
74

SAMPLE STATISTICS - Revision Summary


The exercises in this section have been specifically chosen to underpin the Achievement Criteria that will
be tested in this Achievement Standard.

1. Find the mode and the mean of this data set:


109, 59, 130, 40, 42, 33, 32, 40, 71, 138, 203, 162, 84,
136, 154, 39, 55, 36, 30, 40, 33, 36, 38, 87, 119, 36.

2. The number of nose-to-tail accidents during Labour weekend along a particular stretch of motorway
is given below for each direction. The results have been recorded over the last 2 decades.

North Bound Lanes: 14, 11, 25, 19, 10, 8, 23, 34, 21, 15, 20, 12, 25, 11, 13, 31, 10, 5, 33, 22

South Bound Lanes: 16, 18, 8, 14, 24, 2, 18, 13, 11, 21, 29, 17, 5, 16, 14, 20, 10, 9, 16, 30

a. Create a back to back stem and leaf plot for each data set.
b. List the five point summary for each data set (high, median, quartiles, low).
c. Form box and whisker plots for each on the same grid.
d. Calculate the mean number for each direction.

YEAR 12 MATHEMATICS
Sample Statistics
75

3. For the frequency table:

x 1 2 3 4 5

y 13 9 4 2 2

a. Calculate the mean.


b. Calculate the standard deviation of the data.

4. The cost of a single person to stay one night at a motel in an east coast city varies considerably.
This frequency table shows the tariffs which were charged in 2005.

a. In which interval does the median price lie?


Price Midpoint Frequency
b. Estimate the mean price for a single person to stay of Interval
in a motel. $50 - $55 1
c. Draw a cumulative frequency curve of the data.
$60 - $65 3

$70 - $75 9

$80 - $85 4

$90 - $95 2

$100 - $105 2

$110 - $115 3

$120 - 130 $125 1

YEAR 12 MATHEMATICS
Sample Statistics
76

Each table below has columns to assist the manual calculation of the sample mean and sample standard
deviation.
a. Complete each table, then use the formulae to find each sample statistic.
b. Verify these answers, by using your calculator to find the values.

5. 6.

x x- (x - )2 x f x.f (x - )2 f.(x - )2

3 -5 25 11 2

6 -2 14 8

7 15 13

7 16 16

9 21 9

10 2 22 7

14 6 36 25 5

Totals 56 - Totals

= =

= =

S = S =

= =

YEAR 12 MATHEMATICS
Sample Statistics
77

SAMPLE STATISTICS - PRACTICE TEST 1


Select a sample and use this to make an inference about the population.

THE SITUATION
A newspaper claims that recent significant earthquakes seemed to strike more often in the early hours of
the morning rather than at any other time of the day.

YOUR TASK
Your local newspaper editor has asked you to check this claim and estimate which times of the day of an
arbitrarily chosen year had the most quakes.

YOU NEED TO:


a. Choose a sampling method and use it to obtain a representative sample of at least 30 significant
earthquakes.

b. Describe your sampling process clearly so that someone else can follow it.

c. Justify your choice of sampling method by describing the decisions made and the reason for these
decisions.

d. List the data for your sample that you gathered from the data sheet.

e. Explain whether your sample is actually representative of your population or not. (You do not have
to select another sample if it is not.)

f. Check appropriate statistics for your sample and use this to estimate when in the day there were
significant earthquakes.

g. Write a short paragraph outlining what you have found (from your estimates), and what you could
conclude about when a significant earthquake occurred. Comment on the reliability of your estimate
and therefore your conclusion.

h. Evaluate the sampling and statistical processes you have used. Comment on things such as:

C reliability of your sampling process


C limitations of your sampling process
C the accuracy of your estimate - when an earthquake of this size was most likely
C distribution of the data

You need at least three valid statements.

YEAR 12 MATHEMATICS
Sample Statistics
78

DATA SHEET - Significant Earthquakes of the World - 2005


(of magnitude 6.5 or greater and/or causing fatalities, injuries or substantial damage)
Data from US Geological Survey, Earthquake Hazards Program: http://earthquake.usgs.gov/eqcenter/eqarchives/significant/sig_2005.php
Downloaded 13 Jan 2006.

Date and Time Magnitude Location Date and Time Magnitude Location
Jan 01, 0625 6.7 off West coast of N. Sumatra Jun 04, 1450 6.1 E. New Guinea, PNG
Jan 10, 1847 5.4 N. Iran Jun 06, 0741 5.7 E. Turkey
Jan 10, 2348 5.5 W. Turkey Jun 13, 2244 7.8 Tarapaca, Chile
Jan 12, 0840 6.8 Central-mid Atlantic Ridge Jun 14, 1710 6.8 Aleutian Is, Alaska
Jan 16, 2017 6.6 Yap, Micronesia Jun 15, 0250 7.2 off N. Californian Coast
Jan 19, 0611 6.6 off E. Honshu, Japan Jun 15, 1952 6.5 off Aisen coast, Chile
Jan 23, 2010 6.3 Sulawesi, Indonesia Jun 16, 2053 4.9 greater L.A. area, California
Jan 25, 1630 4.8 Yunnan, China Jun 17, 0621 6.7 off N. Californian coast
Jan 25, 1644 5.9 Turkey - Iraq border Jun 20, 0403 4.7 off W. Honshu, Japan

Feb 02, 0555 4.8 Java, Indonesia Jul 02, 0216 6.6 off Nicaragua Coast
Feb 05,0334 6.6 Anatchan, N.Mariana Is Jul 05, 0152 6.7 Nias Region Indonesia
Feb 05, 1223 7.1 Celebos Sea Jul 05, 1653 2.7 S. Africa
Feb 08, 1448 6.8 Vanuatu Jul 23, 0734 6.0 off S. Honshu, Japan
Feb 14, 2338 6.1 S. Xinjiang China Jul 24, 1542 7.3 Nicobar Is, India
Feb 15, 1442 6.6 Kepulauan, Indonesia Jul 25, 1543 5.0 Heilongjiang, China
Feb 15, 1946 5.5 off S. Honshu, Japan
Feb 16, 2027 6.6 S. mid-Atlantic Ridge Aug 05, 1414 5.2 Yunnan, China
Feb 19, 0004 6.5 Sulawesi, Indonesia Aug 13, 0458 4.8 Yunnan, China
Feb 22, 0225 6.4 Central Iran Aug 16, 0246 7.2 off E. Honshu, Japan
Feb 26, 1256 6.8 Simeulue, Indonesia Aug 21, 0229 5.1 off W. Honshu, Japan

Mar 02, 1042 7.1 Banda Sea Sep 09, 0726 7.7 New Ireland region, PNG
Mar 02, 1112 4.9 Pakistan Sep 24, 1924 5.6 Ethiopa
Mar 05, 1906 5.8 Taiwan Sep 26, 0155 7.5 N. Peru
Mar 09, 1015 5.0 S.Africa Sep 29, 1550 6.7 New Britain region, PNG
Mar 12, 0736 5.7 E.Turkey
Mar 14, 0155 5.8 E. Turkey Oct 01, 2219 5.3 S. Peru
Mar 14, 0943 4.9 Maharashtra, India Oct 08, 0350 7.6 Pakistan
Mar 20, 0153 6.6 Kyushu, Japan Oct 15, 0424 5.2 SW Kashmir
Mar 21, 1223 6.9 Salta, Argentina Oct 15, 1551 6.5 NE of Taiwan
Mar 28, 1609 8.7 N.Sumatra, Indonesia Oct 16, 0705 5.1 E. Honshu, Japan
Oct 20, 2140 5.9 off W. Turkey
Apr 10, 1029 6.7 Kepulauan, Indonesia Oct 27, 1118 4.2 Guangxi, China
Apr 10, 1114 6.5 Kepulauan, Indonesia Oct 29, 0405 6.5 S.E. Indian Ridge
Apr 11, 1220 6.7 N. Coast of N.Guinea
Apr 11, 1708 6.8 S.E. of Loyalty Islands Nov 06, 0211 5.2 Pakistan
Apr 19, 2111 5.5 Kyushu, Japan Nov 08, 0754 5.1 S.China Sea
Nov 14, 2138 7.0 off E.Honshu, Japan
May 01, 1623 4.5 Kyushu, Japan Nov 17, 1926 6.9 Potosi, Bolivia
May 03, 0721 4.9 W. Iran Nov 19, 1410 6.5 Simeulue, Indonesia
May 05, 1912 6.5 S. of Panama Nov 26, 0049 5.2 Huber-Jiangxi, China
May 12, 1115 6.5 Pacific-Antartic Ridge Nov 27, 1022 6.0 S. Iran
May 14, 0505 6.8 Nias Region, Indonesia
May 16, 0354 6.6 S. of Kermadec Islands Dec 02, 1313 6.5 off E. Honshu, Japan
May 19, 0154 6.9 Nias Region, Indonesia Dec 05, 1219 6.8 L. Tanganyila Region, Tanzania
May 23, 0609 4.3 S. Africa Dec 11, 1420 6.6 N. Britain Region, PNG
Dec 12, 2147 6.6 Hindu Kush region, Afghanistan
Dec 13, 0316 6.7 Fiji region
Dec 24, 0201 4.5 W. Honshu, Japan

YEAR 12 MATHEMATICS
Sample Statistics
79

YEAR 12 MATHEMATICS
Sample Statistics
80

YEAR 12 MATHEMATICS
Sample Statistics
81

SAMPLE STATISTICS - PRACTICE TEST 2


THE SITUATION

A marine farm, about 800m offshore, has ropes anchored to the sea floor attached to floating longlines at
the surface. Shellfish grow along each rope and these are shown as numbered spaces on the map (of the
farm layout) below.

1 2 3 4 5 6 7 8 9 10 11 12 13 14 15

30 29 28 27 26 25 24 23 22 21 20 19 18 17 16

31 32 33 34 35 36 37 38 39 40 41 42 43 44 45

214 213 186 185 158 157 130 129 102 101 74 73 46

215 212 187 184 159 156 131 128 103 100 75 72 47

216 211 188 183 160 155 132 127 104 99 76 71 48

217 210 189 182 161 154 133 126 105 98 77 70 49

218 209 190 181 162 153 134 125 106 97 78 69 50

219 208 191 180 163 152 135 124 107 96 79 68 51

220 207 192 179 164 151 136 123 108 95 80 67 52

221 206 193 178 165 150 137 122 109 94 81 66 53

222 205 194 177 166 149 138 121 110 93 82 65 54

223 204 195 176 167 148 139 120 111 92 83 64 55

224 203 196 175 168 147 140 119 112 91 84 63 56

225 202 197 174 169 146 141 118 113 90 85 62 57

226 201 198 173 170 145 142 117 114 89 86 61 58

227 200 199 172 171 144 143 116 115 88 87 60 59

228 229 230 231 232 233 234 235 236 237 238 239 240 241

255 254 253 252 251 250 249 248 247 246 245 244 243 242

YEAR 12 MATHEMATICS
Sample Statistics
82

YOUR TASK

The owners of the marine farm have asked you to help estimate the average mass of shell fish per rope in
the farm.

By selecting a sample of the ropes:

1. Choose a sampling method and use it to design a sampling process to obtain a representative sample
of at least 30 shellfish ropes. Explain your process clearly so that someone else could follow it.

2. Justify your choice of sampling method taking into account the population and considerations of
bias. You may like to describe any decisions you made and the reasons for these decisions.

3. Use your sampling process to select a sample of shellfish ropes.

4. List the data for your sample which you gathered from the data sheet.

5. Explain whether your sample is representative of your population or not. You do not have to select
another sample if it is not.

6. Calculate appropriate statistics for your whole sample.

7. Use your sample and the statistics you have calculated to:
a. Estimate the average mass of shellfish per rope for the whole farm.
b. Estimate a suitable measure of spread for the mass of shellfish per rope for the whole farm.

8. Use your estimate to inform the marine farm owners of your conclusion as to the average mass of
shellfish per rope in the farm. Justify your estimates and therefore your conclusion.

9. Evaluate the sampling process you used. Comment on things like:


C the reliability of your sampling process
C limitations of your sampling process and ways to improve it
C accuracy or appropriateness of your estimate
C the distribution of your data
You need at least 3 valid statements.

YEAR 12 MATHEMATICS
Sample Statistics
83

Table showing mass (in kg) of shellfish on each rope.

1 404 38 488 75 165 112 358 149 159 186 370 223 429
2 413 39 187 76 168 113 269 150 480 187 208 224 446
3 160 40 449 77 508 114 504 151 394 188 139 225 331
4 451 41 397 78 227 115 291 152 257 189 520 226 314
5 337 42 309 79 207 116 189 153 187 190 357 227 180
6 477 43 357 80 186 117 164 154 248 191 499 228 401
7 499 44 255 81 395 118 310 155 378 192 174 229 360
8 186 45 185 82 459 119 195 156 389 193 411 230 219
9 445 46 509 83 191 120 479 157 135 194 504 231 208
10 108 47 185 84 247 121 178 158 407 195 463 232 352
11 216 48 307 85 279 122 484 159 220 196 458 233 267
12 173 49 345 86 468 123 207 160 249 197 285 234 409
13 286 50 175 87 247 124 161 161 281 198 308 235 190
14 191 51 143 88 366 125 252 162 192 199 348 236 167
15 458 52 309 89 336 126 264 163 257 200 240 237 480
16 145 53 365 90 207 127 518 164 294 201 348 238 375
17 425 54 506 91 225 128 210 165 485 202 257 239 385
18 447 55 361 92 275 129 158 166 139 203 381 240 251
19 306 56 490 93 177 130 195 167 460 204 190 241 310
20 327 57 440 94 164 131 338 168 501 205 507 242 428
21 240 58 336 95 458 132 406 169 409 206 214 243 228
22 385 59 359 96 493 133 311 170 350 207 460 244 177
23 438 60 159 97 456 134 411 171 167 208 338 245 201
24 447 61 498 98 168 135 403 172 506 209 464 246 355
25 447 62 291 99 501 136 367 173 298 210 208 247 196
26 405 63 155 100 305 137 490 174 373 211 447 248 265
27 248 64 158 101 413 138 362 175 149 212 191 249 408
28 318 65 429 102 307 139 176 176 334 213 268 250 506
29 165 66 370 103 361 140 195 177 381 214 452 251 177
30 316 67 476 104 489 141 461 178 393 215 295 252 236
31 379 68 257 105 472 142 486 179 232 216 208 253 325
32 354 69 187 106 468 143 348 180 407 217 465 254 297
33 445 70 184 107 175 144 278 181 446 218 260 255 434
34 245 71 344 108 343 145 411 182 160 219 162
35 202 72 440 109 399 146 287 183 155 220 259
36 407 73 187 110 516 147 396 184 274 221 416
37 500 74 410 111 479 148 404 185 362 222 382

YEAR 12 MATHEMATICS
Sample Statistics
84

YEAR 12 MATHEMATICS
Sample Statistics
85

YEAR 12 MATHEMATICS
Sample Statistics
86

YEAR 12 MATHEMATICS
5

The DS-742ET

Mahobe have added some amazing technology into


their new eTool advanced scientific calculator.
• Equation solving.
• Enhanced statistics.
• Improved powers and fraction display.
This calculator is designed to handle even the toughest
assignments. If you use any other calculator then good
luck. With a Mahobe Resource you can have an added
confidence that the answer will be correct.

eTOOL

MAHOBE

www.mahobe.co.nz.
Probability &
87
Normal Distribution

MATHEMATICS 2.6
ACHIEVEMENT STANDARD 90289

Simulate probability situations and apply the normal distribution

Below are the requirements of this Achievement Standard.

C design a simulation process using, for example,


coins, dice or random numbers
From
C use z = (the standard normal
a straightforward
transformation) simulation process

and
C find expected numbers
C apply theoretical techniques using: probability normal distribution
trees, tables, informal conditional probability calculations
C make predictions from simulations
C apply the normal distribution using more than to
one z-value
using theoretical
C find expected numbers from theoretical probability
probability and normal distribution cases
and
C inverse normal distribution problems
the normal distribution
C interpret results from normal distribution cases
and make recommendations in contexts
C relate the results of one simulation to a second requiring several steps
simulation
C combine theoretical and experimental and
probabilities
further interpretation of
C discuss any limitations of the model (or the model and the results.
process) used

YEAR 12 MATHEMATICS
Probability &
88
Normal Distribution

PROBABILITY & NORMAL DISTRIBUTION


- Revision Summary
The exercises in this section have been specifically chosen to underpin the Achievement Criteria that will
be tested in this Achievement Standard.

1. Find the probabilities of each event:


a. Choosing a vowel out of all the letters in the name “WAIKAREMOANA”.
b. Rolling a die and not selecting a prime number.
c. Picking a Jack or a Queen or a King from a shuffled deck of cards (Jokers removed).

2. A wheel is divided into 5 sections labelled A, B, C, D and E.


The wheel is spun around its centre, then a dart is thrown at it.
i. What is the probability that the dart lands in:
a. Region A?
b. Regions C or E?
c. Not in region B?
d. In regions A, B or D?
ii. If the wheel was spun 150 times, how many hits would you expect in region
B?

YEAR 12 MATHEMATICS
Probability &
89
Normal Distribution

3. Two dice are thrown and the sums of the top two faces are recorded.
a. What is the chance that the sum is an even number?
b. What is the probability that the sum is 7?
c. What is the probability that the sum has double digits?
d. What is the probability that the sum is a square number?
e. Find the probability that the sum is a multiple of three.
(First draw a table of possibilities.)

4. Niko is a promising young school athlete who has been training on these throwing events - discus,
shot put and hammer throw for the local championships. He knows that he has a 70% chance of
winning the discus, an 80% chance of winning the shot put and a 75% chance of winning the hammer
throw. The order of the events is as above. Assume the result of each event is independent of the
others.
a. Draw a probability tree of the possible outcomes.
b. What is the probability that Niko wins all three events?
c. Find the probability that Niko wins any two of the three events.
d. Given that Niko wins the first event (discus) what is the chance that he loses the next two?

YEAR 12 MATHEMATICS
Probability &
90
Normal Distribution

5. A paint manufacturer supplies tins of paint which are normally distributed with a mean volume of
4120 ml and a standard deviation of 60ml.
a. What is the probability that a randomly chosen tin of paint has between 4000 ml and 4180 ml?
b. If the manufacturer claims that each tin contains 4 l, what is the chance that a randomly
selected tin has less than 4 l.

YEAR 12 MATHEMATICS
Probability &
91
Normal Distribution

6. Use the Normal Distribution table (page 171) to help answer these questions.
a. Find: b. Calculate:

c. A normal distribution has a mean of 55 and a standard deviation of 8.


Use the conversion formula to find:
i. Pr( x > 61 ).
ii. Pr( 50 < x < 66 ).

YEAR 12 MATHEMATICS
Probability &
92
Normal Distribution

PROBABILITY & NORMAL DISTRIBUTION


PRACTICE TEST 1
Show ALL working.

There are two parts to this activity:

Section A Requires you to design and carry out a simulation to answer questions.

Section B Tests your knowledge of the Normal Distribution. For each question you should write
correct probability statements and show working to support your answer.

These formulae may be useful:

z= or z=

The Good Garden Bag Company provides large bags for people to put their garden clippings into. These
large bags are then collected once a month and replaced by an empty bag. The clippings are tipped into a
truck which compacts then transports the clippings to a composting site.

The collectors on each truck monitor the clippings they pick up - they suspect that 10% of the bags will
have bamboo, while 25% will contain flax. When either bamboo or flax is discovered the homeowner is
given a warning letter which advises them to refrain from placing either of these (forbidden) items in their
bag.

The Good Garden Bag company asks you to investigate the situation. They believe that the presence of
bamboo and flax are independent of each other. (Both bamboo and flax are not wanted since these two
plants do not break down very readily and they may jam the auger at the composting plant).

SECTION A
Design a way to simulate the bag collection of a randomly selected truck, to find out how many of the next
80 homeowners will need to be given the written warning letter.

You need to:


1. a. Describe a method you use in sufficient detail so that another person could repeat it again with
your help.
b. Carry out at least 80 trials of the simulation.
c. Record the result of each trial of the simulation, e.g. in a table.
d. Use the results of your simulation to find the number of homeowners who will receive a letter of
warning for placing these materials in their bags.

YEAR 12 MATHEMATICS
Probability &
93
Normal Distribution

YEAR 12 MATHEMATICS
Probability &
94
Normal Distribution

2. In a normal day a driving team could collect 200 bags.


a. Use theoretical probability to calculate the expected number of households who would receive a
warning letter.
b. Use theoretical probability to calculate how many of the 200 households would be expected to
have both bamboo and flax in their garden bags.
c. Use the results of your simulation to find the expected number of homeowners who should get
the letter if 200 households have their bags collected.
d. Use the results of your simulation to find the expected number of households of the next 200 that
would get a letter for having both flax and bamboo in their bags.

YEAR 12 MATHEMATICS
Probability &
95
Normal Distribution

3. Compare the results of your simulation with the theoretical probability. Make at least one comment
about your simulation.
You could comment on any similarities or differences between the simulation results and the
theoretical probability, or you could comment on ways in which your simulation could be improved
so that it is a better model.

YEAR 12 MATHEMATICS
Probability &
96
Normal Distribution

SECTION B

The operators also gather evidence on the masses of the bags they collect. The maximum limit for each
bag should be 100 kg because manoeuvring these down driveways and paths to a truck can be very
challenging. Analysis of the results showed that the masses of bags were normally distributed with a mean
of 84 kg and a standard deviation of 8.5 kg.

Note: A suitably shaded design or use of proper notation is the minimum working expected.

4. Find the probability that a randomly chosen bag:


a. weighs between 84 kg and 90 kg
b. weighs under 94 kg
c. weighs under the maximum.

YEAR 12 MATHEMATICS
Probability &
97
Normal Distribution

5. a. Find the percentage of bags:


i. which weighed between 100 kg and 105 kg
ii. which weighed over 105 kg, so that the homeowner receives a letter reminding them of the
weight limit.
b. Bags which weigh over 104 kg are monitored for statistical purposes. Out of a 4 day collection of
850 bags, how many would be expected to be over 104 kg?

YEAR 12 MATHEMATICS
Probability &
98
Normal Distribution

6. Very light bags are considered by some drivers as a waste of time, since they still require one person
to collect, replace and load the bag. If 8 bags out of a sample of 175 were found to be “too light”,
use the Normal Distribution and this information to describe a “very light bag” in terms of its mass.

YEAR 12 MATHEMATICS
Probability &
99
Normal Distribution

PROBABILITY & NORMAL DISTRIBUTION


PRACTICE TEST 2
Show ALL working.

Vai is collecting a set of 5 movie character wrist bands which can be found inside packets of Revita cordial
drinks. The manufacturer of Revita experienced some packing machine malfunctions with 30% of the packs
containing 2 wrist bands instead of 1 band.

1. Design a simulation to predict the number of packets of Revita that are required to obtain a full set
of movie character wrist bands. Describe this simulation in sufficient detail so that another person
could repeat it without your help.

YEAR 12 MATHEMATICS
Probability &
100
Normal Distribution

2. Carry out the simulation and record your results.

3. Use your simulation to work out how many packets of Revita are needed on average to get a
complete set.

YEAR 12 MATHEMATICS
Probability &
101
Normal Distribution

4. In Vai’s class there are 28 students altogether (including Vai) trying to collect the 5 wrist bands.
Use the results of your simulation to estimate:
a. How many students will have 4 different wristbands in the set after collecting 5 wristbands.
b. How many students would be expected to have the full set of 5 wrist bands once they have
collected less than or equal to 10 wristbands?

5. After a while, 25 of the 28 students in Vai’s class have only one wrist band to collect to make the
complete set.
a. Use theoretical probability to predict how many of these 25 students will complete their set with
the next packet of Revita.
b. Use theoretical probability to predict how many of these 25 students will complete their set
given that the next packet contains two wrist bands.

YEAR 12 MATHEMATICS
Probability &
102
Normal Distribution

The weights of Revita Packs of cordial are normally distributed with a mean of 315 grams and a standard
deviation of 6 grams.

6. What is the probability that a pack of Revita cordial weighs between 315 grams and 323 grams?

7. What proportion of Revita packets weigh less than 323 grams?

A large carton delivered to a local supermarket contains 950 packets of Revita cordial.

8. How many of these packets would you expect to weigh less than 310 grams?

YEAR 12 MATHEMATICS
Probability &
103
Normal Distribution

9. How many of these packets would you expect to weigh between 310 grams and 321 grams?

10. What is the chance that a packet of Revita cordial will weigh between 321 and 324.5 grams?

11. The manufacturer of Revita regularly checks the machines which weigh the packets of cordial.
Packets under 300 grams or over 325 grams are rejected. What proportion of the packets will the
manufacturer reject?

YEAR 12 MATHEMATICS
Probability &
104
Normal Distribution

12. One day 599 packets are checked and 3 are found to be very light. If the machine is functioning
normally, what is the maximum weight of a very light packet according to this sample?

13. Calculate the mean weight that the machine needs to be set at so that 85% of Revita cordial packets
exceed 308 grams? (Assume the same standard deviation.)

YEAR 12 MATHEMATICS
The DS-742ET

Some advanced technology


has gone into the Mahobe
DS-742ET to make it one
of the most powerful
calculators available.

If you use anything else


then good luck!

planned orbit

eTOOL
actual orbit

C1 C2
www.mahobe.co.nz. MAHOBE
Sequences
105

MATHEMATICS 2.7
ACHIEVEMENT STANDARD 90290

Solve straightforward problems involving arithmetic and geometric


sequences

Below are the requirements of this Achievement Standard.

C find general terms of an arithmetic progression

(AP) Work
C find general terms of a geometric progression
with straightforward cases
(GP) where a, d and r
are evident
C find partial sums of an AP

C find partial sums of a GP


to
C find the sum to infinity, S4 of a GP
situations
C use sigma (3) notation
where other techniques
are used
C manipulate formulae to find a, d or r

C apply the techniques in contexts such as to solve problems

radio active decay, increasing/decreasing % which


and experiments which create sequences
require further
C use logarithmic equations to find n in GP’s interpretation.

C compare sequences

C discuss long term effects from the results

(process) used

YEAR 12 MATHEMATICS
Sequences
106

SEQUENCES - Revision Summary


The exercises in this section have been specifically chosen to underpin the Achievement Criteria that will
be tested in this Achievement Standard.

1. Write the first four terms of the sequence whose nth term is defined by tn = n + .

2. Write down the next two terms of 0, 3, 8, 15, ......, .......

3. Find the first term, a, and the common difference, d, for the arithmetic progression which has
t4 = 4, t5 = 7 and t6 = 10.

4. Find the eighth term and the sum of the first sixteen terms for the arithmetic progression
8, 14, 20, ......, .......

YEAR 12 MATHEMATICS
Sequences
107

5. An Arithmetic Progression has t1 = 8 and t12 = 41.


Find the general term, and the sum of the first 13 terms.

6. How many terms of the series, 7+9+11+13+15+ ...... = 352 (i.e. find n when Sn = 352).

7. For the Geometric Progression 3, 9, 27, ...... find the next two terms, and the eleventh term.

8. A Geometric Progression has a fifth term of -80 and a sixth term of 160. Find an expression for the
general term, tn.

YEAR 12 MATHEMATICS
Sequences
108

9. a. Find the sum to twelve terms of the series 3200 + 1600 + 800 + ......
(Leave your answer as a fraction.)

b. Find the sum to infinity for the same series.

10. Evaluate:

a. 3k + 1

b. (2n - n2)

YEAR 12 MATHEMATICS
Sequences
109

SEQUENCES - PRACTICE TEST 1


Show ALL working.

QUESTION ONE

Carol spends 15 minutes texting her friends on the first day she bought her new mobile phone.
She spends 19 minutes texting on Day 2.
She spends 23 minutes texting on Day 3.
She continues texting daily at the same rate.

a. How long will she spend texting on the sixteenth day?

b. How much time will she spend texting on her new mobile phone over the first sixteen days?

YEAR 12 MATHEMATICS
Sequences
110

QUESTION TWO

Carol’s friend, Tony, spends 24 minutes texting his friends and family on the first day he got his mobile
phone. Each day he increases the time spent texting by 5% from the day before (i.e. he spends 1.05 times
as many minutes as he did the previous day).

What is the total time Tony has spent on his phone in the first 20 days?

QUESTION THREE

The Hong family lease a home security system and make payments every month. The payments reduce
each month by the same percentage. They paid $68 in the second month. They paid $49.13 in the fourth
month and $35.50 in the sixth month and so on.

How much did they pay in the first month?

YEAR 12 MATHEMATICS
Sequences
111

QUESTION FOUR

Tony buys a DVD for $495.


The DVD depreciates at the rate of 26% per year.
How many years to the nearest year, will it take for the DVD to reduce to one-sixth of its original value?

QUESTION FIVE

One of Tony’s uncles helps him to save money by employing him after school. In week 1 he pays Tony $96.
Each week, the uncle reduces the hours and the pay by 17.5% so that Tony can return slowly to full time
study. If this continued indefinitely, how much would Tony’s uncle pay him in total?

YEAR 12 MATHEMATICS
Sequences
112

QUESTION SIX

After several months of using her new mobile phone, Carol decides to reduce her texting by the same
number of minutes each week.

In Week 7, she texted for 595 minutes and by the end of Week 21, her total texting had amounted to
11,235 minutes.

How many minutes did Carol text in Week 1 of her texting reduction plan?

YEAR 12 MATHEMATICS
Sequences
113

SEQUENCES - PRACTICE TEST 2


Show ALL working.

QUESTION ONE

The rungs of a triangular painting trestle decrease uniformly in length. The bottom
rung is 88 cm and each successive rung is 2.75 cm shorter than the previous (lower)
rung. If there are 13 rungs on each trestle, what is the length of the top rung?

QUESTION TWO

A painter, Moe, has just bought some roller blades and is keen to practise as much as he can. On the first
evening after work he roller blades 6 km. Each evening he increases this by 1.25 km more than the
previous evening. If his first day of roller blading was on October 1, what was the total distance Moe would
have roller bladed by the end of October (31 days)?

YEAR 12 MATHEMATICS
Sequences
114

QUESTION THREE

On one of the painting jobs he had, Moe noticed a Yucca plant on the sundeck of the house. The height of
the Yucca was 95 cm and each week he noticed that it grew upwards by another 4% (i.e. 1.04 times taller).
Calculate the height of the Yucca at the end of week 10 (the nearest cm).

QUESTION FOUR

On another job, Moe saw the owner planting bamboo along a border. In week 1 it grew 48 cm after initially
being at ground level. Each week the bamboo’s extra growth length is 12.5% less than the previous week.
What is the maximum height the bamboo will ever grow?

YEAR 12 MATHEMATICS
Sequences
115

QUESTION FIVE

In a park a fountain and statue are surrounded by circular concrete rings.

Moe and his team must repaint the statue (S) with anti-graffiti paint. The concrete rings are centred
around the fountain and have different concrete textures. The percentage increase in area from one ring
to the next is constant. The first ring has an area of 201 m2 while the third ring has an area of 547.22 m2.
What is the area of the outside ring (Ring 4)?

QUESTION SIX

In the same park as the statue and fountain, a retaining wall has been constructed to hold in an earth
bank. Twelve horizontal timber pieces have been used and each is 150 mm shorter than the one below.

If 25.5 metres of timber was used altogether what must the length of the lowest piece be?

YEAR 12 MATHEMATICS
Sequences
116

QUESTION SEVEN

Moe plans to set up his own painting company so he has been looking for a tidy, recent model, second hand
van to transport his equipment to jobs. He sees one that will be perfect and notes that there are two
possible payment regimes.

PLAN A pay a deposit of $4950


first month pay $300
each successive month pay $20 more than the previous month

PLAN B pay a deposit of $7995


first month pay $615
each successive month pay $10 less than the previous month

Calculate how many months it will take before the total paid into Plan A would be the same amount paid
into Plan B.

YEAR 12 MATHEMATICS
5

The DS-742ET

Mahobe have added some amazing technology into


their new eTool advanced scientific calculator.
• Equation solving.
• Enhanced statistics.
• Improved powers and fraction display.
This calculator is designed to handle even the toughest
assignments. If you use any other calculator then good
luck. With a Mahobe Resource you can have an added
confidence that the answer will be correct.

eTOOL

MAHOBE

www.mahobe.co.nz.
Trigonometry Problems
117

MATHEMATICS 2.8
ACHIEVEMENT STANDARD 90291

Solve trigonometry problems requiring modelling of practical situations

Below are the requirements of this Achievement Standard.

C take measurements in a practical situation


with suitable calculations to follow From
C calculations could include use of the Sine Rule,
measurements taken
Cosine Rule and/or areas of triangles
lead to
C contexts to explore could be bearings,
straightforward
relative velocity, etc calculations
C find length and angles using: Sine Rule
to
and/or Cosine Rule
C find circular measures using: arc length and/or more complex
sector areas trigonometry situations

where
C find triangular areas
suitable models

C use 2 dimensional representations of and rules are selected


3 dimensional situations
resulting
C combine any of the above techniques
to solve more integrated contextual problems in sensibly rounded
solutions (in context)

with appropriate units.

YEAR 12 MATHEMATICS
Trigonometry Problems
118

TRIGONOMETRY PROBLEMS
- Revision Summary
The exercises in this section have been specifically chosen to underpin the Achievement Criteria that will
be tested in this Achievement Standard. (Note, diagrams have not been drawn to scale.)

1. Calculate the lengths or angles that have been marked.


a. b. c.

2. Use the sine rule or cosine rule to find the missing lengths.
a. b.

3. Use the sine rule or cosine rule to find the missing angles.
a. b.

4. a. Convert these to radians. Give your answer to 2 decimal places.


i. 30E ii. 165E iii. 238E

b. Change these to degrees. Give your answer to 1 decimal place.

i. 1.06 rad ii. rad iii. 5.89 rad

YEAR 12 MATHEMATICS
Trigonometry Problems
119

YEAR 12 MATHEMATICS
Trigonometry Problems
120

5. Calculate the marked dimensions in these sectors.


a. b. c.

(Find θ in radians and degrees)

6. Find the areas of these figures.


a. b. c. d.

Find the shaded area.

7. Three fisherman are angling along the banks of a river. Two of them, F and G, are on the same bank
and are 50 metres apart. The third, H, is on the opposite bank of the river. It is known that pHFG is
62E and angle pFGH is 66E. How wide is the river estuary?

YEAR 12 MATHEMATICS
Trigonometry Problems
121

PRACTICAL TRIGONOMETRY PROBLEMS


PRACTICE TEST 1
Show ALL working.

This model assessment is in two parts.

PART A
1. You and some other students go to a local park near your school. On a level piece of ground your
teacher has marked out a large four sided area. You have been given tape measures, magnetic
compasses, trundle wheel and alidade.

2. From a reference point, X, inside this quadrilateral, you must begin your measurements. Note that
the point X is not allowed on any sides nor any diagonals of the quadrilateral.

3. You draw the quadrilateral and include point X.

4. Using the equipment, from point X, complete a radial survey. Indicate the measurements you will
need to take to calculate the area of the quadrilateral and mark these on your diagram.

NOTE
Usually you need to draw your diagram from scratch. If you use the diagram given, the highest grade
you will be awarded is ‘Achievement’. For the purpose of continuing PART A, assume that you have
drawn the shape below (to scale) and it is 1: 1500.

YEAR 12 MATHEMATICS
Trigonometry Problems
122

YEAR 12 MATHEMATICS
Trigonometry Problems
123

PART B

Use the sketch and your measurements from PART A to complete this section of the task. You do not need
any other measurements to complete this.

1. Council drainage contractors are to check the old pipe which lies diagonally across the quadrilateral
in Part A. Calculate the shortest distance between the pairs of opposite corners (i.e. the lengths of
both diagonals of the quadrilateral).

2. A memorial rose garden is going to be set up in the North-West corner of this quadrilateral. To assist
with the planning, you are asked to find the size of the interior angle in that corner of the
quadrilateral.

YEAR 12 MATHEMATICS
Trigonometry Problems
124

3. As the contractors explore the old drainage pipe, they find that there is a thick layer of sludge and
sediment which covers the bottom of the pipe.

The radius of the pipe (OP or OQ) is known to be 1.25 m.

a. If the distance across the top of the sediment (i.e. chord ) is


2.24 m, then calculate the area of the cross-section of the
sediment (shown by the shaded area on the diagram).

b. Assume this measurement is the average width of the sediment in


the pipe. Also assume that the pipe runs along the longer of the
two diagonals you calculated in #1.
What volume of sediment is the pipe holding?
(Round this number to the nearest 10 units.)

YEAR 12 MATHEMATICS
Trigonometry Problems
125

PRACTICAL TRIGONOMETRY PROBLEMS


PRACTICE TEST 2
Show ALL working.

This model assessment is in two parts.

PART A
1. As part of a measuring task, a class of Year 12 students met their teacher at the local ice skating
rink. There they were paired up and given the measuring task, with a measuring tape and, of course,
ice skates!

An arbitrary point, Q, on the blue line was given to them by the teacher. They had to measure and
record the lengths needed to find the angle pLQR. (Every student pair had a different location for
Q.)

A triangle is set with Q at one corner, on the blue line, and the other two corners, L and R, as the
goal posts.

NOTE: If a pair of students couldn’t identify the measurements which they needed to take, they
were supplied with a help sheet. This usually has the diagram of the triangle ªLQR,
supplied with labels L, R and Q.

If students use the help sheet, then the highest grade which they can be awarded for
this task is ‘Achievement’.

At the end of the session the measurements are usually handed in to the teacher.

YEAR 12 MATHEMATICS
Trigonometry Problems
126

PART B

1. For the position given (to one pair of students) and the measurements provided, calculate the angle
in the triangle on the blue line, pLQR.

2. If a student was standing at point Q, looking towards the goal mouth, what would the “apparent
width” of the posts be to her? i.e. find the length on the diagram.

(Note: ªRXQ is not right angled but isosceles.)

YEAR 12 MATHEMATICS
Trigonometry Problems
127

3. a. A student, Wayne (W) is 4.29 m from the right goal post, R. The distance between the goal posts
(LR) is 1.83 m. The bearing of the left post, L, from Wayne is 093E
and the bearing of the right post, R, from Wayne is 118E.

Another student, Sheena (S) is standing further along the goal line,
leaning on the rink wall.

What is the bearing of Sheena (S) from the left goal post?
Note - round all angles to a nearest degree.

b. How far is Wayne from the left goal post?

YEAR 12 MATHEMATICS
Trigonometry Problems
128

4. Near each goal line there are two “face off” spots. Each “face off” spot is the centre of a circle
with a 4.57 metre radius.

A sponsor usually paints their logo or one of their products inside these
circles before big games are played. One sponsor who imports sports
gear has painted a sports helmet inside the circle, and centred it at
the “face off” spot, O.

The angle (below) pAOC = 2.17 radians, while AO = OC = 3.36 metres.

Calculate the area inside the circle which has not been painted.

YEAR 12 MATHEMATICS
Trigonometry Problems
129

5. Just as the students are completing their measurements, an ice hockey team arrives for a training
session. The class decide to watch them practise. During one of the drills a player attempts to flick
the puck into the back of the net. (The height of the goal is 1.22 m.)

The player stands to the side of the goal mouth, so that the puck is:
3.5 metres from the goal line.
4.9 metres from the near post.
5.3 metres from the far post.

The player aims the puck at a point, T, which is 30 cm


below the top of the centre of the crossbar.

Find the angle of elevation pTPC.

Note - in any triangle ªABC then AB2 + AC2 = 2AD2 + 2BD2

YEAR 12 MATHEMATICS
The DS-742ET

Some advanced technology


has gone into the Mahobe
DS-742ET to make it one
of the most powerful
calculators available.

If you use anything else


then good luck!

planned orbit

eTOOL
actual orbit

C1 C2
www.mahobe.co.nz. MAHOBE
Trigonometry Problems
130

MATHEMATICS 2.9
ACHIEVEMENT STANDARD 90292

Solve straightforward trigonometry equations

Below are the requirements of this Achievement Standard.

C solve trigonometric equations using


trigonometric graphs in degrees within the From
range 0E # θ # 360E and in radians within
straightforward
the range π # θ # 2π
(simple step)
C solve trig equations such as:
equations
5 sin 3x = 4 on [0E, 360E]
cos (x - 180E) = 0.5 on [-180E, 180E] to
2 sin x - 1.5 = -1 on [0, 2π]
equations requiring
using degrees or radians as directed several steps

in contexts
C solve trigonometric equations in contexts
such as tidal motion or wheel revolutions or

situations involving
C manipulate more complex trigonometric
equations more challenging
manipulations.

C solve problems in contexts, including


mathematical situations

YEAR 12 MATHEMATICS
Trigonometric Equations
131

TRIGONOMETRIC EQUATIONS
- Revision Summary
The exercises in this section have been specifically chosen to underpin the Achievement Criteria that will
be tested in this Achievement Standard.

1. a. Change these degree measures to radians (to 2 dp).


i. 30E ii. 114E iii. 268E

b. Write each degree measure as a fraction of π.


i. 30E ii. 105E iii. 320E

c. Change these radian measures to degrees (to 1 dp where necessary).

i. 0.92 radii. 2.63 rad iii. rad

YEAR 12 MATHEMATICS
Trigonometry Problems
132

2. a. Draw i. y = 2 sin x.
and ii. y = cos x - 1, on separate graphs.

On graph i. use 0 # x # 360E and on graph ii. use 0 # x # 2π.

b. On the first graph, draw the line y = 1.6 and find x where 2sin x = 1.6.

On the second graph, draw the line y = and find x where cos x - 1 = .

YEAR 12 MATHEMATICS
Trigonometric Equations
133

3. Solve these equations with a suitable method.


a. 2 sin x = 1, on 0E # x # 360E
b. 5 cos x - 2 = 1 on 0E # x # 2π
c. cos 2x = 0.85 on 0 # x # 2π
d. 2.5 sin 3x = -1.5 on 0E # x # 360E
e. sin (x - 45E) = 0.4 on -270E # x # 270E

YEAR 12 MATHEMATICS
Trigonometry Problems
134

4. On the interval 0 # x # 4π, which values of x would provide the greatest and least values

of y = 5 sin ?

YEAR 12 MATHEMATICS
Trigonometric Equations
135

TRIGONOMETRIC EQUATIONS
PRACTICE TEST 1
Show ALL working.

QUESTION ONE

Solve the following trigonometric equations:


a. cos x = 0.4, 0E # x # 360E

b. sin x + 4 = 3.09 on 0E # x # 360E

YEAR 12 MATHEMATICS
Trigonometry Problems
136

c. 3 tan x = 4.8 on 0 # x # 2π

QUESTION TWO

Solve cos 2x = 0.78, 0E # x # 360E.

YEAR 12 MATHEMATICS
Trigonometric Equations
137

QUESTION THREE

Millie hears a “click - clack” sound while riding her bike. She dismounts
and finds a small tack has embedded itself into the rubber tread.

She knows that her bike wheel has a 66 cm diameter and once she is
home, she turns it and observes how the tack on the tyre rotates around
the central wheel hub.

The height (H) of the tack relative to the centre hub may be given by:
H = 33 sin (45t)E

H = height in centimetres.
t = time in seconds after the wheel begins to rotate.

a. After how many seconds will the tack first be 30 cm above the hub?
b. How long will it take the tack to return to its starting position?

YEAR 12 MATHEMATICS
Trigonometry Problems
138

QUESTION FOUR

While watching her piano being tuned, Millie notices a tuning fork for the note Middle C. The pure tone of
the tuning fork when struck, vibrates at 256 times per second.

Millie believes these vibrations may be modelled by the trigonometric function y = 0.6 sin (512 π t), where
the amplitude (loudness) is 0.6.

The speed of the vibrations suggests the number of cycles per second cannot be detected by us.

What is the least amount of time taken for the tuning fork to reach a loudness of 0.45?

YEAR 12 MATHEMATICS
Trigonometric Equations
139

QUESTION FIVE

Over a long period of time, Millie has observed the depth (d) of water at Flint’s Stone, a large rock near
the entrance to the local harbour. She believes that the tidal cycle is about 12½ hours. At low tide the
water depth beside the rock is only 1.5 metres, but at high tide it has reached 6.1 m.

Millie believes this formula, d = 2.3 cos + 3.8, could model the depth around Flint’s Stone,

where: d = depth of the water in metres.


t = time in hours after high tide.

Fishing boats may only enter the harbour when the water there is at least 3 metres deep. How long either
side of high tide, can boats safely travel in and out of the harbour?

YEAR 12 MATHEMATICS
Trigonometry Problems
140

TRIGONOMETRIC EQUATIONS
PRACTICE TEST 2
Show ALL working.

Solve the following trigonometric equations:

QUESTION ONE

a. cos θ = 0.4, 0E # x # 360E

b. 2.5 sin θ + 1 = 1.5, 0E # x # 360E

c. tan θ + 2.7 = 1.1, 0 # θ # 2π

YEAR 12 MATHEMATICS
Trigonometric Equations
141

QUESTION TWO

Solve the following trigonometric equations:

a. cos (θ - 30E) = -0.7, 0E # x # 360E

b. sin 2θ = 0.61, 0 # θ # π

YEAR 12 MATHEMATICS
Trigonometry Problems
142

QUESTION THREE

A patient with a fever is admitted to hospital for further observation. Her temperature varies from a low
of 37E to a high of 40.2E Celcius.

A model of the temperature has been suggested as: T = 38.6 + 1.6 sin where t is the time in days.

During which day, following her admission to hospital, does the patient’s temperature first reach 40E C?

YEAR 12 MATHEMATICS
Trigonometric Equations
143

QUESTION FOUR

The eleven weeks from the middle of July until early October is a profitable time for a local winter sports
retailer to sell snow boards. Sales of snow boards have been monitored over the same period of time for
some years. They have developed this trig formula to model the numbers of snowboards that they sell over
the 11 weeks.

S = 30 - 33 cos , where week 1 # t # week 11.

According to the formula over how many weeks would they sell at least 45 snowboards?

YEAR 12 MATHEMATICS
Answers
144

THE ANSWERS
Note: the answers have been checked and rechecked. If your answer differs from the one listed then check with your
teacher, friends or write to Mahobe Resources. Due to space restrictions, we have not been able to set out all answers
as fully as we would have liked. Remember - in the exam you should put each step of your answer on a separate line.

Pages 6-10, ALGEBRA - Revision Summary QUESTION THREE


a. By substituting, equations become
1. a. x = b. r = = x2 + (2x + 6)2 = 36
x2 + 4x2 + 24x + 36 = 36
c. x = or x = -11 d. x = 0, , -8 5x2 + 24x = 0
x(5x + 24) = 0
2. a. 12 - 9x b. 18x3 - 24x2 - 10x,
c. 4x3 - 16x2 - 35x + 147 x = 0 or x = -
3. a. (x - 12)(x - 7) b. (5x + 7)(2x - 3)
b. If x = 0, y = 6, ˆ first is (0, 6)
c. (a - v)(a + w)
If x = - , y = -3.6, ˆ second is (-4.8, -3.6)
4. a. b. 3x3y5 c. m6

5. a. 35 = 243 QUESTION FOUR


b. i. log 5, ii. log 72, iii. log 30
c. 2 =

6. a. 3x - 1 b. (x - 12)(1350 + 24x)= (1800 + 20x)(x - 9)


24x2 + 1062x - 16200 = 20x2 + 1620x - 16200
c. a + b d. 2n+10 4x2 - 558x = 0
7. a. x = -50 b. n = 2x(2x - 279) = 0
ˆ x = 0 or x = 139.5
Check: x = 139 tickets, PA < PB
c. x < 1 d. x >-
x = 139.5 tickets, PA = PB
x = 140 tickets, PA > PB
8. a. x = 7, x = -6 b. x = ,x=2 ˆ sell 140 tickets or more, price B is cheaper.

c. x = - , x = 0, x = 1 d. x = 1.15, x = -0.65 QUESTION FIVE

e. x = 0.45, x = -4.45 f. x = -3.37, x = 2.37 Rule: α + β = , αβ =


7
9. a. x = 3 = 2187 b. x = =7 Let roots be α and α + 1
c. x = 2.67 ˆ sum of roots, k + 1 = α + α + 1
10. a. (-4, 3) b. (-2, 3) i.e. k + 1 = 2α + 1
c. (2, -2) and (3, 0) d. (-2, -1) and (1, 2) and k = 2α (A)
11. a. i. ª = 57 > 0 ˆ 2 real distinct roots Product of roots, 4k = α (α + 1)
ii. ª = 0 ˆ 1 real (repeated) root 4k = α2 + α (B)
b. (2n)2 - 4(2)(5) < 0 Y n2 < 10 ˆ - <n< ˆ from equation (A) α =

Pages 11-14, ALGEBRA PRACTICE TEST 1 substituting this into equation (B)
QUESTION ONE
1. x - 5 = 4k
2. 3x4
3. log 8 = 4k
4. a. x = b. x = 2.13 (2 dp) c. x = or x = -1 k2 + 2k = 16k
k2 - 14k = 0
QUESTION TWO
k(k - 14) = 0
M = M0(0.85)t
ˆ possible values: k = 0 or k = 14
500 = 800(0.85)t
Check: if k = 0, x2 - x = 0 Y x(x - 1) = 0
= 0.85t k = 14, x2 - 15x + 56 = 0
Y (x - 7)(x - 8) = 0
0.625 = 0.85t
log 0.625 = t log (0.85)
t =

t = 2.89 (2 dp) days


= 3 days

YEAR 12 MATHEMATICS
Answers
145

Pages 15-18, ALGEBRA PRACTICE TEST 2 QUESTION SIX


QUESTION ONE
First Instance (A) V =
1. 12x3 + 101x2 + 37x - 24

2. Second Instance (B) V + 250 =

Substituting (A) into (B)


3. or 1.2
+ 250 =
4. a. x = 3, b. x = or x = 2, c. x = 1.92
multiply by t(t - 1)
5. 7c + 2p = 1925 (t - 1)5000 + 250 t(t - 1) = 5000t
c = p + 50 5000t - 5000 + 250t2 - 250t = 5000t
by substitution, 7(p + 50) + 2p = 1925 250t2 - 250t - 5000 =0
9p + 350 = 1925 t2 - t - 20 =0
9p = 1575 (t - 5)(t + 4) =0
p = 175 t = 5h (positive choice)
ˆ plain ice creams cost $1.75 ˆ in first instance, t= 5h, V = ,

QUESTION TWO = 1000 km/hr


By substitution
in second instance, t = 4h, V =
x2 + (x - 3)2 + 2x - 7 =0
x2 + x2 - 6x + 9 + 2x - 7= 0 = 1250 km/hr
2x2 - 4x + 2 = 0
2(x2 - 2x + 1) = 0 Pages 20 - 24, GRAPHS - Revision Summary
(x - 1)2 = 0 1. a. i. y = x2 + 3x - 4
x =1
By back substitution, y = -2
there is only one co-ordinate set (1, -2)
ˆ only 1 point of intersection

QUESTION THREE
Set up 2 equations: h×w = 1.7
h =w+1
(w + 1)(w) = 1.7
w2 + w - 1.7 =0
By quadratic formula

w=

w= or

w = 0.896 or -1.896
Ignoring the negative result
width = 0.9 m (1dpf) and height = 1.9m (1dp)

QUESTION FIVE ii. y = (x + 3)2 - 4


of 19995 = 13330

ˆ solve 13330 = 19995(0.993)t with =

ˆ = (0.993)t

log = log (0.993)t

log = t log (0.993)

=t

t = 57.7 h
or t = 58h (2 sf)

YEAR 12 MATHEMATICS
Answers
146

Pages 20 - 24 cont c. x2 + y2 = 49
1. a. iii. (x + 1)(x - 2)(x + 3) (x + 2)2 + (y - 1)2 = 49

b. i. y=
d. y = 5x
y = 5x - 3

e. y = log8 x
y = log8 |x - 3 |

ii. y =

2. a. y = (x + 5)(x + 1)(x - 2)
b. y = (x - 1)2 - 3
= x2 - 2x - 2
c. y = 5x - 2
d. (x - 3)2 + (y + 2)2 = 25

YEAR 12 MATHEMATICS
Answers
147

Pages 20 - 24 cont c. y = 4x
3. Using a table of values

x y

-3 0.0156

-2 0.0625

-1 0.25

0 1

1 4

2 16

3 64

Pages 25 - 30, GRAPHS - PRACTICE TEST 1


1. a. y = x2 - 4x - 5
= (x - 5)(x + 1)
x intercept (5, 0), (-1, 0), y intercept (0, -5)
QUESTION TWO
Vertex x = (5 + -1) ÷ 2
1. Features can include:
y = (2 - 5)(2 + 1)
< The two graphs meet at one point, (1, 3)
ˆ (x, y) = (2, -9)
< y + x = 4 is a straight line
< y = x2 - 3x + 5 is a parabola
< y + x = 4 is a tangent to the curve y = x2 - 3x + 5
< y = x2 - 3x + 5 does not meet the x axis but does
intersect the y axis at (0, 5)
< Vertex of the curve y = x2 - 3x + 5 is (1.5, 2.75)
2. a.
t V

0 $11 955

1 9 836

2 8 065

3 6 614

4 5 423

5 4 447
b. y = Using a table of values
6 3 647

x -4 -2 -1 1 2 4

y -1 -2 4 4 2 1

b. During the fourth year (t . 3.5 y)

YEAR 12 MATHEMATICS
Answers
148

Pages 25 - 30 cont QUESTION SIX


QUESTION THREE a. Use the 2nd and 3rd lines of the data table with the
a. formula to attain two simultaneous equations.
15.2 = 8.0 =

15.2(15 - b)= a 8.0(66 - b) = a


228 - 15.2b =a 528 - 8.0b = a

ˆ 228 - 15.2b = 528 - 8.0b


8.0b - 15.2b = 528 - 228
- 7.2b = 300
b = -41.7 (1 dp)
or b = -42 (2 sf)

If using b = -41.7, then a = 528 - 8(-41.67)


= 861.6
ˆ D =

b. y = If using b = -42, then a = 864


ˆ D =
=3+

x = 0, y = -2 (0, -2) b. Using D =


y = 0, 3x + 2 = 0 (-2/3, 0)
VA: x = 1 HA: as x 6 0, y 6 3 6.6 =

6.6(t + 41.7) = 861.6


6.6t + 275.2 = 861.6
6.6t = 586.4
t = 88.8h
ˆ 89 hours after the initial time (after 4pm, 10 Feb)
i.e. 3 days, 17 hrs later, 9am on 14 February.

Pages 31- 38, GRAPHS - PRACTICE TEST 2


QUESTION ONE
a. intercepts at x = 0, x = 1, x = -3 or use a table

x -4 -3 -2 -1 0 1 2

y -20 0 6 4 0 0 10

QUESTION FOUR
a. In the early years, the rate decreases the car value the
quickest i.e. it loses half its value, one third its value,
one quarter its value etc. In later years the value changes
less rapidly.
b. The car loses its value less rapidly in later years with the
value only changing marginally. The remaining value stays
above $1000.
c. The y intercept tells us that the car was bought for just
under $12,000 (i.e. $11, 995)

QUESTION FIVE
a. y = (x + 2)(x - 1)(x - 4) b. vertex at (-1, 4)
or y = x3 - 3x2 - 6x + 8 intercepts (0, 3), (-3, 0) and (1, 0) or use a table
b. x2 + y2 = 25
c. HA: y = 2, VA: x = -3
x -4 -3 -2 -1 0 1 2
Intercepts (1.5, 0) and (0, -1)
ˆ y= or y = 2 - y -5 0 3 4 3 0 -5

YEAR 12 MATHEMATICS
Answers
149

Pages 31- 38, (cont) y = 2x2 - 3x - 5


QUESTION ONE
b.

c. b. y = -x3 + 1

d. Features to include:
< diameter = 6 units ˆ radius = 3 units
< circle centre = (1, 0) QUESTION FOUR
< equation is (x - 1)2 + y2 = 32 Using a table of values:
< x intercepts at y = 0, (-2, 0) and (4, 0)
< y intercepts at x = 0
x -5 -4 -3 -2 -1 0 1 2 3 4
(-1)2 + y2 = 32
y2 = 9 - 1 y 128 64 32 16 8 4 2 1
y = ˆ at (0, ) (0, )

QUESTION TWO
a. Equation y = (x - 2)2 or y = x2 - 4x + 4
b. y = 2x
c. xy = 6 or y =

QUESTION THREE
a. Solve the equation first
2x2 - 3x - 5 = 0
(2x - 5)(x + 1) = 0
ˆ x = 2.5, x = -1
Intercepts (-1, 0), (2.5, 0) and (0, -5)
QUESTION FIVE
Vertex: x = a. Initial investment was $4 500
b. Since 4500(1.08)9 = $8995.52 is very close to double the
ˆ x = initial $4 500 then the end of year 9 is close.
ˆ during year 10 the sum would double.
c. At the end of year 12 there is $11 332 (nearest dollar) in
ˆ y = 2( )2 - 3( )-5
the fund. If $3000 was withdrawn and assuming no penalty
clause (early withdrawal) with the same interest rate as
= -6 before, the graph would plunge by $3 000, before
continuing to compound from the new balance of $8 332
ˆ vertex = ( , -6 )

YEAR 12 MATHEMATICS
Answers
150

Pages 31- 38, (cont)


4. = 2x + 5
QUESTION SIX
a. gradient 2+5 = 7
equation at (1, 6) is y - 6 = 7(x - 1)
or y = 7x - 1
or 7x - y - 1 = 0
b. when x = 0, y = 0
equation at (0, 0) is y = 5x
5. = 2x - 3

gradient = 1 Y 2x - 3 = 1
ˆ x =2
substituting x = 2 Y y = 4 - 6 + 2
=0
ˆ at (2, 0), gradient = 1
6. a. = 125 - 27
= 98
b. A quick sketch helps

a. Using the two pieces of information, then equating with


each other.
Week 1: 107449 = A - B log(1 + 0.5)
Week 10: 64011 = A - B log(10 + 0.5)
Making A the subject of each equation:
107 449 + B log(1.5) = 64 011 + B log (10.5)
107 449 - 64 011 = B log (10.5) - B log (1.5)
43 438 = B log

= B log 7 (2x - x2) dx =

=B
= [4 - ] - [0]
ˆ B = 51 400 (nearest 10)
Using Week 1 A = 107 449 + B log (1.5)
= units2
= 107 449 + 51 400 log (1.5)
= 116 500 (nearest 10) c. y = x3 + 2x2 - 3x
Check A in other equation
A = 64 011 + 51 400 log(10.5)
= 116 500 T
ˆ Model equation for spending is:
S = 116 500 - 51 400 log(x + 0.5)
b. If x = 30 then S = 116 500 - 51 400 log(30.5)
= $40 207 (nearest $)

Pages 40 - 44 CALCULUS - Revision Summary


1. a. = 27x8 - 10x
ˆ Area = | y . dx | + | y . dx |
b. f(x) = 6x3 - 10x2 + 21x - 35
ˆ f!(x) = 18x2 - 20x + 21
2. a. f(x) = x3 + 4x2 - 11x + c = | (x3 + 2x2 - 3x)| + | (x3 + 2x2 - 3x)|dx
b. y! = 18x5 + 6x-4 + 1
ˆ y = 3x6 - 2x-3 + x + c
= x4 + x3 - x2 + x4 + x3 - x2
or y = 3x6 - +x+c

3. a. f!(x) = 4x3 - 10x + 1 = + - + 0-( - 18 - )


ˆ f!(2) = 32 - 20 + 1
= 13 = +
b. = 2x - 2, ˆ gradient = -2 - 2
= 11 units2
= -4

YEAR 12 MATHEMATICS
Answers
151

Pages 40 - 44 cont Pages 45 - 49, CALCULUS - PRACTICE TEST 1


QUESTION ONE
7. a. = 3x2 - 3
= 3x2 - 6, ˆ gradient = 3(5)2 - 6
b. turning point is when =0
= 69
2
ˆ 3x - 3 = 0
3(x2 - 1) = 0 QUESTION TWO
3(x + 1)(x - 1) = 0
x =±1 A = (3x2 + 1) dx
substituting back into original equation y = x(x2 - 3)
x = 1, y = -2 and x = -1, y = 2 and using cubic shape,
= x3 + x + c
ˆ (-1, 2) is a local maximum
(1, -2) is a local minimum = [ 23 + 2 ] - [ 13 + 1] c cancels out
c. y = x3 - 3x increases before the max i.e. x < -1 = 10 - 2
and after the minimum i.e. x > 1 = 8 units2
decreases between the max and min
i.e. -1 < x < 1 QUESTION THREE
8. h = 245t - 4.9t2 If f!(x) = 6x2 - 4x + 5
a. v = then f(x) = (6x2 - 4x + 5) dx

= 245 - 9.8t = 2x2 - 2x2 + 5x + c


b. v(5) = 245 - 9.8(5) At (2, 11), f(x) = 11 and x = 2
= 196 m/s ˆ 11 = 2(2)3 - 2(2)2 + 5(2) + c
c. h(5) = 245(5) - 4.9(5)2 11 = 16 - 8 + 10 + c
= 1102.5 m c = -7
ˆ curve is f(x) = 2x3 - 2x2 + 5x - 7
d. = - 9.8 m/s2
QUESTION FOUR
(since no t variable is present, a is constant
Parallel to the x-axis implies the gradient = 0
at -9.8 m/s2)
e. h is a maximum when it turns, i.e. the turning point is ˆ = 6x2 - 6
when v = 0 ˆ solve 245 - 9.8t = 0
gradient = 0 when 6x2 - 6 = 0
t = 6(x2 - 1) = 0
6(x + 1)(x - 1) = 0
when t = 25s it is at maximum height,
ˆ x = -1 or x = 1 are the x coordinates
and h(25) = 245(25) - 4.9(25)2
= 3062.5 m above the ground.
QUESTION FIVE
9. 1. First identify the two formulae The gradient of the function is = 3x2 - 6x - 7

y at x = -1, gradientm = 3(-1)2 - 6(-1) - 7


=2
x equation at (-1, 4) is y - 4 = 2(x - -1)
y-4 = 2x + 2
Perimeter is 2x + 2y = 108 y = 2x + 6
Area is xy (we need to optimise this) or 2x - y + 6 = 0
2. Use the perimeter formula and make one of the
variables the subject QUESTION SIX
2y = 108 - 2x d = v . dt
y = 54 - x
3. Substititute this whole expression into the Area. = 0.75 t . dt
This leaves just one variable to contend with.
A = xy d = 0.75 +c
= x (54 - x)
= 54x - x2 since t = 0 and d= 0 (i.e. vehicle hasn’t moved from the
4. Differentiate starting point) then c = 0
A! = 54 - 2x ˆ d = 0.75 or d = 0.375t2
5. Solve A! = 0 to find maximum or minimum
54 - 2x = 0 after 8 seconds d = 0.375(8)2
x = 27m d = 24m (moved from start of roll)
6. Substitute back into original equation
y = 54 - 27
= 27m
Since x = y = 27m, the rectangle is a square.
Maximum area is 272 = 729 m2

YEAR 12 MATHEMATICS
Answers
152

Pages 45 - 49 (cont) Pages 50 - 54, CALCULUS - PRACTICE TEST 2


QUESTION SEVEN QUESTION ONE
The curve would be translated 36 metres upwards
a. = 4x3 - 6x
from to ˆ gradient = 4(2)3 - 6(2)
= 32 - 12
= 20
b. y = (8x3 + 6x2 - 4x - 1) dx
= 2x4 + 2x3 - 2x2 - x + c
At (-1, 1) y = 1 when x = -1
ˆ 1 =2-2-2+1+c
ˆ c =2
ˆ y = 2x4 + 2x3 - 2x2 - x + 2

c. A = (x3 + 2) dx

= + 2x + c

= +4 - 0
2 2
ˆ Area = (12 - 3x + 36) dx = 8 units
d. If y = x-2, = -2x-3 or

= (48 - 3x2)dx
ˆ =

= 48x - x3 + c x3 =
= [128] - [47]
= 81 m2 (Area of the parking bay )
x3 = -8
QUESTION EIGHT x =
First find some labels for dimensions x = -2
ˆ y =
y y
= ˆ Coordinates are (-2, )
x
QUESTION TWO
x + 2y = 5.6 or x = 5.6 - 2y
To maximise the cross sectional area,
A1 + A2 = ( x2 - 3x - 4) + ( x2 - 3x - 4) dx
A = xy
= (5.6 - 2y)y
= 5.6y - 2y2 = - x2 - 4x + - x2 - 4x

= 5.6 - 4y = - 24 - 16 - 0

= 0 when 5.6 - 4y = 0 + - - 20 - - 24 - 16

ˆ y = = -18 + -15 - -18


y = 1.4 m
substituting back into original equations = 21 units2
x = 5.6 - 2 × 1.4
x = 2.8 m
A = xy
= 2.8 × 1.4
= 3.92 m2

YEAR 12 MATHEMATICS
Answers
153

Pages 50 - 54 (cont) 3. Make y the subject


QUESTION THREE 2y - 3x = 7
a. d = (18 + 15t - 3t2) dt 2y = 3x + 7
y = x+ ˆm=
= 18t + t2 - t3 + c
4. Using the points (-3, 2) and (1, 5)
when t = 2 s, d = 65 cm
ˆ m =
65 = 18(2) + (2)2 - 23 + c

65 = 36 + 30 - 8 + c =
7 =c
ˆ equation is y - 5 = (x - 1)
ˆ d = 18t + t 2 - t3 + 7
4(y - 5) = 3(x - 1)
when t = 0 (stationery), d = 7 cm from owner
4y - 20 = 3x - 3
3x - 4y + 17 = 0
b. If v = 18 + 15t - 3t2
5. Either use 2x + 5y - 8 = 0 or y = x+
the = 15 - 6t (acceleration)
a. Parallel line, 2x + 5y + c = 0
maximum velocity is when a = 0
Using (1, -2) 2 - 10 + c = 0
i.e. 15 - 6t = 0
c =8
t = ˆ 2x + 5y + 8 = 0
b. perpendicular line 5x - 2y + d = 0
t = 2.5s (swap coefficients, reverse the sign)
ˆ maximum velocity Using (5, 6) 25 - 12 + d = 0
v = 18 + 15(2.5) - 3(2.5)2 d = -13
= 36.75 cm/s ˆ 5x - 2y - 13 = 0
6. Solving simultaneously
QUESTION FOUR a. [3x + 4y = 10]× 5 Y [15x + 20y = 50]
V = 335, V = πr2h [5x + 3y = 13]× 3 Y -[15x + 9y = 39]
ˆh= 11y = 11
y =1
S = 2 πr2 + 2 πrh Back substitute 3x + 4(1) = 10
= 2 πr2 + 2 πr . 3x = 10 - 4
x =
S = 2 πr2 + 670
To optimise, differentiate the surface area function. x = 2 ˆ (2, 1)
Check 5(2) + 3(1) = 13 T
ˆ = 4 πr - 670
b. [0.1x - 0.2y - 0.7 = 0]× 40 Y [4x - 8y = 28]
[0.4x + 0.3y - 0.6 = 0]× 10 Y - [4x + 3y = 6]
0 = 4πr - -11y = 22
y= ˆ y = -2
= 4πr
Back substitute 0.1x + 0.2(-2) - 0.7 = 0
= r3 0.1x + 0.4 - 0.7 = 0
0.1x = 0.3

=r x =

r = 7.30 cm (3 sf), (h = 2.00 cm) ˆ x = 3 and coordinate = (3, -2)


Check 0.4(3) + 0.3(-2) - 0.6 = 0 T
Pages 55 - 60, CO-ORDINATE GEOMETRY 7. A quick sketch can assist.

1. a. Mid point = (2, 8)

b. = (1.6, -0.8)

2. a. d =
=
= 7.2 (s 2f)
b. d =
=
= 14.7 (1 dp)

YEAR 12 MATHEMATICS
Answers
154

Pages 55 - 60 (cont) Pages 61 - 66, COORDINATE GEOMETRY


PRACTICE TEST 1
7. a. Midpoint of = QUESTION ONE
a. d =
= (2, 3 )
=
=
Equation of median, m = d = 19.1 grid squares
ˆ true distance = 19.1 × 20 m
= 382 m
= b. gradient =

= =

Gradient of median y+1 = (x + 3) equation is (y - -2) = (x - -10)

10(y + 1) = 9(x + 3) 14(y + 2) = 13(x + 10)


ˆ 9x - 10y + 17 = 0 ˆ 13x - 14y + 102 = 0
c. The fence line has the same gradient as the given
b. Altitude is a perpendicular line from (1, 5) to equation ˆ y = x+c
Gradient of , m =
using (-2, 8) 8 = (-2) + c
=
8 = -1 + c
ˆ c =9
=
ˆ fence equation is y = x+9
ˆ gradient of z line =
QUESTION TWO
= -2
The drain line is a perpendicular bisector of the line which
Equation of z line y - 5 = -2(x - 1)
joins the two points.
ˆ 2x + y - 7 = 0
c. perpendicular bisector cuts at the midpoint Midpoint =

Midpoint of = (-1, 2) = (2, -3)


Gradient between the two points, m =
Gradient of =
= -1
= ˆ the bisector gradient must be mz = 1
ˆ equation is (y - -3) = 1(x - 2)
y+3 =x-2
ˆ gradient of perpendicular =
x-y-5 =0
or y =x-5
Equation of perpendicular bisector is y - 2 = (x + 1)
QUESTION THREE
Using the midpoint 3(y - 2) = -2(x + 1)
The median joins N(4, 11) to the midpoint of ST
ˆ 2x + 3y - 4 = 0
8. Compare lengths of all three sides.
If two sides equate thenªOPQ is isosceles midpoint =

= (-4, -1)
Length =
gradient =
=

=
Length =
= =
=
equation y - 11 = (x - 4)
Length =
= 2(y - 11) = 3(x - 4)
2y - 22 = 3x - 12
Since the length of = length ˆ 3x - 2y + 10 = 0 is the equation of the median
then ªOPQ is isosceles

YEAR 12 MATHEMATICS
Answers
155

QUESTION FOUR QUESTION TWO


Firstly, find where the altitude line intersects SP
The midpoint of T1 T3 =
x + y + 12 = 0
x- y - 2=0 = (6, 11)
2x + 10 = 0
2x = -10 Length from C to L3 =
x = -5 =
= 13.04 m (2 dp)
Back substituting x + y + 12 = 0
-5 + y + 12 = 0 QUESTION THREE
y = -7 Altitude through T2 must be perpendicular to the other
ˆ the altitude meets line SP at point Q (-5, -7) side of the triangle T3H2
ˆ the length of the altitude must be the distance
m of T3H2 =
between Q(-5, -7) and T(2, 0)
d = =
=
= 9.899 grid units ˆ Mz =
grid lines are 20m apart therefore multiply by 20
= 198 m (3 sf) the equation through T2(25, 15) is
y - 15 = (x - 25)
QUESTION FIVE
Find a line through (6, 0) which is perpendicular to 2(y - 15)= -5(x - 25)
y= x + 8, mz = 2y - 30 = -5x + 125
5x + 2y - 155 = 0
ˆ equation is y = x+c
QUESTION FOUR
at (6, 0), 0 = -8 + c ˆ c = 8 First find the gradients and the equations of each line

ˆ perpendicular line through (6, 0) is: y = x+8 Gradient of T1H1 is =

The point of intersection of these two lines appears to be ˆ Mz =


(0, 8) since c = 8 in both line formulae
ˆ the closest distance is between (0, 8) and (6, 0) At (9, 0) equation of perpendicular
d = y-0 = (x - 9)
=
4y = 27 - 3x
= 10 units (× 20)
3x + 4y = 27
= 200 m

Gradient of BH3 is = -1, ˆ Mz = 1


Pages 67 - 72, COORDINATE GEOMETRY
- PRACTICE TEST 2 At (3, 15) equation of perpendicular:
QUESTION ONE y - 15 = 1(x - 3)
x - y = -12
a. L2 = Therefore coordinates of D must be the intersection of
the two legs. Solve these simultaneously.
= (18, 21 ) x - y = -12 6 4x - 4y = -48
3x + 4y = 27 6 3x + 4y = 27
b. m = 7x =-21
x = -3
= Back substitute y = -3 + 12
y=9
ˆ coordinates of D are (-3, 9)
equation is (y - 15) = (x - 25)
QUESTION FIVE
8(y - 15) = -3(x - 25)
ˆ 3x + 8y - 195 =0 Midpoint of T3 B is = (5 , 25)
c. If parallel, then m = , ˆ equation is y = x+c
This point will be on a perpendicular line from T3T2
From Question 1b the equation is 3x + 8y - 195 = 0
26 = (10) + c
ˆ gradient of T3T2 is =
26 - 4 = c
c = 22 ˆ mz =
ˆ parallel line is y = x + 22

YEAR 12 MATHEMATICS
Answers
156

Pages 67 - 72 (cont)
equation of this line (y - 25) = (x - 5 )

3(y - 25) = 8(x - 5 )

or 8x - 3y + 31 = 0
Lines meet at closest point to new light:
solve 8x - 3y = -31 6 64x - 24y = -248
3x + 8y = 195 6 9x + 24y = 585
73x = 337
x = 4.616 (3 dp)
Back substituting 8y = 195 - 3 × 4.616
y = 22.644
ˆ coordinates are (4.616, 22.644)
ˆ Distance between new light and existing cable
5. Totals: 3x = 56, 3(x - 0 )2 = 72, 0 = 8, s = 3.464
= 2.516 (3 dp) 6. Totals 3f = n = 60, 3xf = 1053, 3f(x - 0 )2 = 832.85
ˆ 2.5 m is the closest distance 0 = 17.55, s = 3.757

Page 77 - 78 SAMPLE STATISTICS, PRACTICE TEST 1


Pages 74 - 76, 2.5 SAMPLE STATISTICS Note, this is a possible solution only.
- Revision Summary Sampling process:
1. Modes are 36 & 40, Mean is = 76.23 • Assign each member of the population (of 87 earthquakes)
a two digit number 00 - 86.
• Using your calculator’s random function, start at, say, the
2. a. Northern Southern (8 + 7) = 15th number.
4 3 13 0 • Choose the second and third digits of every three digit
5 5 3 2 1 02 0 1 4 9 random number.
954321100 1 01344666788 • Disregard numbers which are > 86.
85 0 2589 • Ignore repeated numbers (®).
b. Nth 5 (Low), 11 (LQ), 17 (Med), 24 (UQ), 34 (High) • Stop when you have a list of approx 30 quakes.
Sth 2 (Low), 10.5* (LQ), 16 (Med), 19 (UQ), 30 (High)
* should really be 10-11 accidents Number Date Time R
c. Box and whisker plot for Labour Weekend nose to Magnitude

tails. 17 19/02 00 04 6.5


06 23/01 20 10 6.3
Northern Direction 46 14/06 17 10 6.8
76 14/11 21 38 7.0
51 20/06 04 03 4.7
96 - - -
02 10/01 23 48 5.5
Southern Direction
57 25/07 15 43 5.0
18 22/02 02 25 6.4
36 03/05 07 21 4.9
41 19/05 01 54 6.9
0 10 20 30 40 58 05/08 14 14 5.2
51 ® - - -
d. Northern mean = 18.1 (18 accidents) 75 8/11 07 54 5.1
Southern mean = 15.55 (15+ accidents)
46 ® - - -
3. a. 0 = 2.033 (3 dp)
64 26/09 01 55 7.5
b. s = 1.217 (3 dp)
(Compare σ = 1.196) 84 12/12 21 47 6.6
4. a. n = 25, the 13th price lies in the $70 - interval 34 19/04 21 11 5.5
b. 0 . $85.40 91 - - -
c. Tarif Cum Freq 93 - - -
$50 0 44 06/06 07 41 5.7
$60 1 93 - - -
$70 4
95 - - -
$80 13
93 - - -
$90 17
$100 19 59 13/08 04 58 4.8
$110 21 63 24/09 19 24 5.6
$120 24 55 23/07 07 34 6.0
$130 25

YEAR 12 MATHEMATICS
Answers
157

Number Date Time R The evenness of this distribution would suggest that quakes
Magnitude can occur at any time including the early hours of the
81 02/12 13 13 6.5 morning and the latest hours of the night. Though random
28 21/03 12 23 6.9 sampling may be time consuming, we return to the theme,
59 ® - - - that ignoring the vagaries of calculator random programming,
57 ® - - - every earthquake in the population had the same chance of
14 15/02 14 42 6.6 selection for the sample.
Pearson developed a measure of skewness of a distribution.
37 05/05 19 12 6.5
72 27/10 11 18 4.2 Skew =
65 29/09 15 50 6.7
08 25/01 16 44 5.9 =
02 ® - - -
= -0.528
52 02/07 02 16 6.6
99 - - - Since the co-efficient of skewness generally lies between -3
94 - - - and +3, this suggests that the distribution of the sample times
76 ® - - - tend to negative skewness, so a few early hours are pulling
64 ® - - - the mean away from the median.
83 11/12 14 20 6.6
13 14/02 23 38 6.1 Page 81 - 83 SAMPLE STATISTICS, PRACTICE TEST 2
Note, this is a possible solution only.
03 12/01 08 04 6.8
The layout of the shellfish farm suggests that an ‘all in’
Though it is physically possible to test all the items in this sample may not provide the most representative 30 selections
population of earthquakes, it is not essential, as a sample of from the population. One possibility is to look at five possible
about 30 is a useful representation of the population. Also, strata.
with the random function on the calculator (though it is not
truly random) we are generally satisfied that each quake in
the population had the same chance of selection in the
sample. This simple random sampling method, though reliant
on the generated random numbers, has provided 30
selections. If doubts rise over its authenticity, the exercise
could readily be repeated with another simple random
sample.

If this sample of 30 could be represented in a stem and leaf


plot these sample statistics of earthquake times would result.
LQ = 07 21, UQ = 19 12, median = ½ way between
13 13 hours and 14 14 hours (13 43 ½)

The box and whisker plot of this sample would look like:
Based on these proportions:
Box Plot of Earthquake Times During Day The top row should have × 30 .5 in the sample
(sample n = 30)
The left group should have × 30 .7 in the sample

The middle group should have × 30 .7 in the sample

The right group should have × 30 .8 in the sample


From this sample it would appear that the times are skewed
slightly to the later end of the day with a medium time of The bottom end should have × 30 .3 in the sample
13 : 43 : 30
30 (Total)
The mean of the sample 0 . 12 26 and the standard deviation
of the sample . 7hr 18 min. This would support the above Take a simple random sample of each strata / group in the
result (box plot) with both the mean and median appearing as population. Note each site is already numbered
p.m. times for this sample. e.g. Top end:
Assign the first two digits from the random number
The distribution of the time in this sample appears to have a generator of your calculator starting at (e.g. 4+5) 9th
reasonably even spread, i.e. number.
00 h - 09 h 12 quakes Disregard any numbers > 45 or < 01
10 h - 19 h 12 quakes Ignore repeated numbers (®).
20 h - 23 h 6 quakes Stop when we have 5 for this strata.

YEAR 12 MATHEMATICS
Answers
158

Page 81 - 83 (cont) Visually this Box and Whisker Plot of the sample is:
e.g. 62 37 68 78 14 34 94 68 82 86 99 89 63 49 34 02 29
ˆ 37, 14, 34, 02, 29 for the top end
Apply the same technique (using 3 digit numbers) to each of
the other four strata, with sensibly applied numbering to suit
each case. The goal is to finish with a total sample size of at
least 30 shellfish sites

Example of possible strata selections: The distribution is skewed with the median placed in the
Top End (001-045) upper end of the interquartile range. The middle 50% of
# Mass (kg) the sample has a mass between 240 kg and 407 kg.
37 500 The mean of the sample, 0 = 335.7 kg
14 191
The sample standard deviation, s = 114.0 kg
Using Pearson’s skewness rule:
34 245
02 413 Skew =
29 165
Left Group (045 - 115) =
77 508
= -0.889
59 359
The data is negatively skewed with the smaller masses
49 345 pulling the mean away from the median. The masses of
107 175 the shellfish lines are quite spread out. Both mean and
47 185 median point towards the mid 300 kg mass, so if the
109 399 owners of the farm were looking for a conservative
88 366 estimate of the mass of their crop, they should tend
Middle Group (116 - 171) towards the lower statistic of the two.
155 378
Another sample could produce a similar result, or not, but
160 249
the exercise could readily be repeated. It is important to
158 407 consider the sites of the areas and which are closer to
145 411 sea, to shore, to nutrients, affected more by weather etc.
148 404 Therefore the strata sampling would still be a sensible
137 490 option for this farm.
156 389
Right Group (172 - 227) Pages 88 - 91, PROBABILITY AND NORMAL DISTRIBUTION
Revision Summary
178 393
227 180 1. a. b. c. =
189 ® 520
180 407 2. a. = b. = c. =
174 373
196 458 d. = e. 150 × = 25 times
188 139
3. Table of possibilities
200 240
+ 1 2 3 4 5 6
Bottom End (228 - 255) 1 2 3 4 5 6 7
233 267 2 3 4 5 6 7 8
254 297 3 4 5 6 7 8 9
230 219 4 5 6 7 8 9 10
5 6 7 8 9 10 11
It is physically possible to have used the whole 6 7 8 9 10 11 12
population, but it is too time consuming, and a sample
such as this has attempted to represent the whole a. = b. = c. =
population. Also, with the areas of the shellfish farm split
into strata, each sub-area is catered for and the
d. Pr( 4 or 9) =
randomness of the selection process, again is dependent
on the generated random number function from the
calculator. e. Pr (3 or 6 or 9 or 12) = =

The sample provides these statistics when ordered.


LQ = 240 kg, median = 369.5 kg, UQ = 407 kg

YEAR 12 MATHEMATICS
Answers
159

Pages 88 - 91 cont
= Pr ( <z< )
4. a.
= Pr(-0.625 < z < 1.375)
= Pr (0 < z < 0.625) + Pr(0 < z < 1.375)
= 0.2340 + 0.4155
= 0.6495

Pages 92 - 98, PROBABILITY & NORMAL DISTRIBUTION


PRACTICE TEST 1

1. a. One possible method and the suggested answers using


this method.
Use a 3 digit random number generator from a
calculator. Assign the first digit in the random number
b. P(WWW) = 0.7 × 0.8 × 0.75 to bamboo.
= 0.42 e.g. let a 1 in the first digit = bamboo present, (being
c. P(any two W) = WWL or WLW or LWW 1 out of 10 or 10%)
= 0.14 + 0.105 + 0.18 ˆ 0, 2-9 in the first digit position = bamboo absent
= 0.425 Assign the second and third digits (as a pair) for flax
d. P(loses last two / wins first) e.g. let pairs 00 to -24 = flax present (f), (being 25 out
= 0.2 × 0.25 of 100 or 25%) pairs -25 to -99 = flax absent
= 0.05 A table of this simulation based on the above is given.
5. 0 = 4120 ml, s = 60 ml
RAN# RAN# RAN# RAN# RAN# RAN#

745 - 754 - 854 - 097 - 259 - 636 -

155 B 348 - 643 - 247 - 970 - 632 -

580 - 012 F 791 - 685 - 331 - 550 -

a. Pr(4000 < V < 4180) = 13.5 + 34 + 34 355 - 292 - 211 F 761 - 630 - 671 -
= 81.5%
= 0.815 175 B 728 - 867 - 515 F 511 F 445 -

b. Pr(V < 4000) = 2.5% 033 - 744 - 954 - 850 - 773 - 257 -
6. a. Pr(0 < z < 1.6) = 0.4452
b. Pr(-2.14 < z < 1.16) 763 - 127 B 944 - 557 - 801 F 926 -
= Pr(0 < z < 2.14) + Pr( 0 < z < 1.16)
556 - 923 F 719 F 814 F 423 F 303 F
= 0.4838 + 0.3770
= 0.8608 795 - 237 - 541 - 744 - 739 - 986 -
c. 0 = 55, s = 8
301 F 464 - 460 - 121 B,F 007 F 688 -

938 - 683 - 592 - 790 - 167 B

583 - 730 - 623 F 189 B 863 -

421 F 793 - 329 - 361 - 865 -

684 - 254 - 569 - 315 F 897 -


i. Pr(x > 61)
= Pr ( z > ) d. Letter given to 21 households
= Pr( z > 0.75)
2. One possible approach (apart from recognising
= 0.5 - Pr ( 0 < z < 0.75)
independence) is a tree diagram.
= 0.5 - 0.2734
= 0.2266
ii. Pr(50 < x < 66)

YEAR 12 MATHEMATICS
Answers
160

Pages 92 - 98 (cont)
Pr( <z< ) = Pr(1.882 < z < 2.471)
a. P(get letter) =
= 0.025 + 0.075 + 0.225 = 0.4932 - 0.4700
= 0.325, = 0.0232
ˆ expect 200 x 0.325 = 65 letters = 2.32%
b. P(both) = 0.025
ii. Pr(x > 105) = Pr(z > )
ˆ expect 0.025 x 200 = 5 would have both
c. From the simulation, 21 out of 80 would get a letter = Pr(z > 2.471)
ˆ expect 200 x = 52 = 0.5 - 0.4932
= 0.0068
ˆ 52 - 53 households are to receive a letter. = 0.68%
d. From the simulation, 1 out of 80 had both.
b. Pr(x > 104) = Pr(z > )
ˆ expect 200 x =2
= Pr(z > 2.353)
ˆ 2 - 3 households with both = 0.5 - 0.4907
3. Note: Example comment only. The simulated results have = 0.0093
lower values than the corresponding theoretical results. Expect 850 x 0.0093 = 7.905 , i.e. 8 bags
The random feature of a calculator is not truly random, 6.
but generated therefore a second sample of 80 numbers
could have more than the theoretical values. Also there is
no allowance in the theoretical case whether an operator
who is monitoring these items, actually misses them in a
particular bag (or bags)(i.e. human error).

Section B
4. µ = 84 kg, σ = 8.5 kg
Use inverse normal
i.e. Pr(0 < x < W) = 0.4543
ˆ Pr(0 < z < ) = 0.4543

ˆ = -1.688 (it is on the negative side)

or using x = σz + µ
a. Pr(84 < x < 90) = Pr(0 < z < ) = -1.688 x 8.5 + 84
= 69.65 kg
= Pr(0 < z < 0.706) = under 70 kg
= 0.2598
b. Pr(0 < z < ) + 0.5 Pages 99 - 104, PROBABILITY & NORMAL DISTRIBUTION
PRACTICE TEST 2
Pr(0 < z < 1.176) + 0.5 = 0.3802 + 0.5 Possible Simulation
= 0.9802 1. 70% of the packs will have 1 wristband ˆ use random
numbers 000 - 699
30% of the packs will have 2 wristbands ˆ use random
numbers 700 - 999
The packs with 2 wristbands could have 15 possibilities,
i.e. Band 1 & Band 1 etc
1&1 2&2 3&3 4&4 5&5
1&2 2&3 3&4 4&5
1&3 2&4 3&5
c. Pr(x < 100) = Pr(0 < z < ) + 0.5
1&4 2&5
= Pr(0 < z < 1.882) + 0.5 1&5
= 0.4700 + 0.5 Assume that the bands are randomly selected before
= 0.97 being packed. For the pairs with one wristband, assign
5. a. i. Pr(100 < x < 105) these random numbers (700 ÷ 5 = 140 possibilities)
ˆ Band 1 000 - 139
Band 2 140 - 279
Band 3 280 - 419
Band 4 420 - 559
Band 5 560 - 699

YEAR 12 MATHEMATICS
Answers
161

Pages 99 - 104 cont Section B


For the packs with two wristbands, assign these random 6. µ = 315 g, σ = 6 g
numbers (300 ÷ 15 = 20 possible pairs).
(1&1) 700-719 (2&2) 800-819 (3&3) 880-899
(1&2) 720-739 (2&3) 820-839 (3&4) 900-919
(1&3) 740-759 (2&4) 840-859 (3&5) 920-939
(1&4) 760-779 (2&5) 860-879 (4&4) 940-959
(1&5) 780-799 (4&5) 960-979
(5&5) 980-999
2. Here is a possible simulation exercise based on the above.
Pr(315 < x < 323) = P( 0 < z < )

RAN# RAN# RAN# RAN# RAN# RAN# = P(0 < z < 1.333)
= 0.4087
406 3 347 3 991 5,5 814 2,2 737 1,2 969 4,5 7. P(x < 323) = 0.5 + 0.4087
= 0.9087
307 3 065 1 507 4 679 5 637 5 665 5
8. P(x < 310)
301 3 938 3,5 596 5 715 1 459 4 578 4

297 3 836 2,3 246 2 662 5 227 2 223 2

135 1 112 1 597 5 740 1,3 021 1 494 4

3. Ten packets were needed in this simulation example to


get all the wrist bands (see above).
4. a. After five bands (packets), based on this simulation P(z < ) = P(z < -0.833)
were collected so no student would have the four
= 0.5 - P(0 < z < 0.833)
different bands.
= 0.5 - 0.2975
b. After ten bands(packets), based on this simulation, = 0.2025
ˆ 950 x 0.2025 . 192 packets
were collected every student would have a full set. 9. P(310 < x < 321)
5. The one band packs (5 possible choices) in 70% or (14%
each band) and the two band packs (15 possible choices)
in 30% (or 2% each pair) provide a useful picture if we
choose one numbered band
The diagram is of part of the tree diagrams.
e.g. if 4 is the missing band, there is a 24% chance (from
14 + 10) that the next packet will have a 4.

P( <z< ) = (-0.833 < z < 1)

= 0.2975 + P(0 < z < 1)


= 0.2975 + 0.3413
= 0.6388
ˆ 950 x 0.6388 . 607 packets
10. P(321 < x < 324.5)

a. ˆ 25 students x 24% = 5.76 or almost 6 students P( <z< ) = (1 < z < 1.5383)


b. If the next packet is a double band packet, then 10%
is possible = 0.4433 - 0.3413
ˆ 25 students x 10% = 2.5 or 2 - 3 students = 0.102

YEAR 12 MATHEMATICS
Answers
162

Pages 99 - 104 (cont)


S16 = [2a + (n - 1)d]
11. P(300 < x < 325)

= [ 2 x 8 + 11 x 6]

= 8[16 + 66]
= 656

5. T12 a + 11d = 41
P( <z< ) = P(-2.5 < z < 1.667) T1 a = 8
11d = 33
= 0.4938 + 0.4522 ˆ d=3
= 0.946
ˆ reject 1 - 0.946 = 0.054 or 5.4% ˆ s13 = [ 2 x 8 + 12 x 3]
12. P(0 < x < W) = 0.495
= [16 + 36]

= 338
6. a = 7, d = 2
Sn = [ 2 x 7 + (n - 1) x 2]

352 = [ 14 + 2n - 2]
or P(0 < z < ) = 0.495
352 = [12 + 2n]
= - 2.576
352 = 6n + n2
or W = 6 x -2.576 + 315 n2 + 6n - 352 = 0
“Very light” packets have w = 299.5 g (1 dp) (n + 22)(n - 16) = 0
ignore n = -27, ˆ n = 16 (terms)
13. P(0 < z < ) = 0.35
7. 3, 9, 27, 81, 243; ˆ a = 3, r = 3
ˆ t11 = a x r10
= 3 x 310
= 177 147
8. T6 a x r5 = 160
T5 a x r4 = -80

by long division Y r = -2

ˆ = -1.036 ˆ if t6 = 160, then a x r5 = 160


a x (-2)5 = 160
308 = µ - 1.038 x 6
308 + 6.228 = µ a=
µ = 314.228
a = -5
New mean = 314 (3 sf)
ˆ Tn = (-5) x (-2)n - 1 is the general term for this GP
9. 3200 + 1600 + 800 + ...., ˆ a = 3200, r =
Pages 105 - 108, SEQUENCES - Revision Summary
a. s12 =
1. T1: 1 + = 2, t2: 2 + =2 , t 3: 3 + =3

T 4: 4 + =4 =

ˆ 2, 2 ,3 ,4
= 6398
2. 24, 35
3. ...., ...., ...., 4, 7, 10 ˆ d = 3
a =4-3-3-3
= -5 b. S4 =
4. a = 8, d = 6, t8 = a + (n - 1)d
=8+7x6 =
= 50

= 6400

YEAR 12 MATHEMATICS
Answers
163

Pages 105 - 108 cont QUESTION FIVE


k+1 2 3 4 5 6
a = 96, r = (100 - 17.5)%
10. a. 3 =3+3 +3 +3 +3 +3 = 82.5% or 0.825
= 1092 ˆ S4 =
2
b. (2n - n )= 1 + 0 - 3 - 8 - 15 - 24 - 35 - 48 = $548.57 (2 dp)

= -132 QUESTION SIX

Pages 109 - 112, SEQUENCES - PRACTICE TEST 1 After 21 weeks, Sn = [ 2a + (n - 1) x d]


QUESTION ONE
a. a = 15, d = 4, Tn = a + (n - 1)d S21 = [2a + 20d]
T16 = 15 + 15 × 4
= 75 min = 21a + 210d
= 11 235
=1 hours
In week 7, Tn = a + (n - 1)d
T7 = a + 6d
b. Sn = [ 2a + (n - 1)d] = 595
Solving simultaneously to find d, then a
S16 = [ 2 × 15 + 15 × 4] 21a + 210d = 14 235 6 21a + 210d = 11235
a + 6d = 595 6 21a + 126d = 12495
= 8 [ 30 + 60 ] 84d = - 1260
= 720 min (or 12 hours) d = -15
i.e. it reduces by 15 minutes each week
QUESTION TWO To find a, T7: a + 6d = 595 minutes
Sn = where a = 24, r = 1.05 a + 6 × -15 = 595
first week a = 685 minutes
ˆ S20 = (i.e. 24 + 25.2 + 26.46 + ...) Pages 113 - 116, SEQUENCES - PRACTICE TEST 2
QUESTION ONE
= 793.6 minutes (or 13 hours, 13.6 minutes)
a = 88, d = -2.75, n = 13
ˆ Tn = a + (n - 1)d
QUESTION THREE
T13 = 88 + 12 × -2.75
T4 a x r3 = 49.13
= 55 cm (length of top rung)
T2 a x r1 = 68.00

by division = QUESTION TWO


a = 6, d = 1.25, n = 31
ˆ r2 = 0.7225 ˆ r = 0.85 Sn = [ 2a + (n - 1)d]
since r = then T 2 = r × T1
S31 = [ 2 × 6 + 30 × 1.25]

and T1 =
= [12 + 37.5]

= = 767.25 km (Total distance travelled)

= $80 (first monthly payment) QUESTION THREE


a = 95 cm, r = 1.04, n = 10
QUESTION FOUR t10 = a × r9
a or T1 = $495, 100% - 26% = 74% (or 0.74) = 95 × 1.049
ˆ Tn = × 495 = 135.2
= 135 cm (nearest cm)
= $82.50
ˆ T n = a × rn - 1 QUESTION FOUR
= 82.50 If growth in week 1 is T1 = 48 cm, r = (100 - 12.5%)
solving 495 × 0.74n - 1 = 82.50 = 87.5%
= 0.875
0.74n - 1 =
As n 6 4, ˆ S 4 = 48 + 42 + 36.75 + 32.156 ...

(n - 1) log(0.74) = log ( ) =

= 384 cm (or 3.84 m)

n-1 =

n - 1 = 5.95
n = 6.95 ( near the end of 7 years)

YEAR 12 MATHEMATICS
Answers
164

Pages 113 - 116, cont Pages 117 - 120, TRIGONOMETRY PROBLEMS


QUESTION FIVE Revision Summary
T1 = a = 201, T3 = a × r2 1. a. x2 = 8.62 - 7.22
= 547.2 x =
= (from ) x = 4.7 m (2 sf)
b. sin 62 =
r2 = 2.722
r = 1.65 or 165%
ˆ x =
ˆ Area of fourth ring
A4 = 201 × 1.653 x = 126 cm (3 sf)
= 902.9 m2
c. cos θ =
QUESTION SIX
a = ?, n = 12, d = -150 mm, S12 = 25 500 mm θ = cos-1
ˆ Sn = [2a + (n - 1)d]
θ = 32.0E 1 dp

25500 = [2a + 11 × -150] 2. a. x2 = 1.142 + 1.062 - 2 × 1.14 × 1.06 × cos 61


= 2.4232 - 2.4168 × cos 61
25500 = 6[2a - 1650]
25500 + 9900 = 12a x =
12a = 35400 x = 1.12 m (2 dp)

a = b. =

ˆ lowest piece = 2950mm or 2.95m x =

QUESTION SEVEN x = 59.8 mm (3 sf)


First, compare some early figures for each payment plan.
3. a. cos θ =
Plan A: Deposit $4950, M1 = 300, M2 = 320, M3 = 340...
Plan B: Deposit $7995, M1 = 615, M2 = 605, M3 = 595 ...
ˆ equate the two schedules by summing each then θ = cos-1
combining them as an equation.
θ = 72.9E
SA: 4950 + [ 2 × 300 + (n - 1)20]
b. =
= 4950 + [580 + 20n]
θ = sin-1
2
= 4950 + 290n + 10n
θ = 49.1E (1 dp)
4. a. i. 0.52 rad ii. 2.88 rad iii. 4.15 rad
SB: 7995 + [2 × 615 + (n - 1)(-10)] b. i. 60.7E ii. 75E iii. 337.5E
5. a. l = 5.6 × 1.11
= 7995 + (1240 - 10n) = 6.2 cm (1 dp)

= 7995 + 620n - 5n2 b. l = 114 × 48E ×


SA = SB
4950 + 290n + 10n2 = 7995 + 620n - 5n2
= 95.5 mm (3 sf)
15n2 - 330n - 3045 = 0
15(n2 - 22n - 203) = 0 c. θ =
( n - 29) (n + 7) = 0
ignore n = -7, ˆ n = 29 (monthly payments) = 1.63 rad (2 dp)
Check 6. a. A = × 64 × 26
Plan A: 4950 + (2 × 300 + 28 × 20) = 21770
= 830 cm2 (2 sf)

Plan B: 7995 + (2 × 615 + 28 × -10) = 21770 b. A = × 5.8 × 13.4 × sin 52

= 30.6 m2 (1 dp)
c. A = × 2452 × 0.72

= 21 600 mm2 (3 sf)


d. A = A1 - A2

= × 34.52 × 95 × - × 34.52 × sin 95

= 394 cm2 (3 sf)

YEAR 12 MATHEMATICS
Answers
165

Pages 117 - 120, cont PART B


7.

=
1. Diagonal AC p BXD = 48E + 113E
= 161E
x = By Cosine Rule
AC2 = 1232 + 136.52 - 2 × 123 × 136.5 × cos 161E
x = 56 m (2 sf)
Then AC =
AC = 256 m (3 sf)

= sin 66

d = 56 × sin 66
d = 51 m (2 sf)

The river is 51 m wide at this point.

Pages 121 - 124, PRACTICAL TRIGONMETRY PROBLEMS


PRACTICE TEST 1
PART A

Diagonal BD; p BXD = 48E + 113E


= 161E
By Cosine Rule
BD2 = 1352 + 722 - 2 × 135 × 72 × cos 161E
BD =
BD = 204 m (3 sf)
ˆ shortest distance is BD = 204 m

2. North West corner is pBAD = pBAX + pDAX

Area of each ª
ªABX = ab sin x

= × 123 × 72 × sin 113E

= 4080 m2 (3 sf)
ªBCX = × 72 × 136.5 × sin 86E

= 4900 m2 (3 sf)
ªCDX = × 136.5 × 135 × sin 113E

= 8480 m2 (3 sf)
ªDAX = × 135 × 123 × sin 113E

= 6170 m2 (3 sf)
TOTAL Area = 23,630 m2 (or 2.363 ha)

YEAR 12 MATHEMATICS
Answers
166

Pages 121 - 124 cont 2.


By Cosine Rule
AB2 = 1232 + 722 - 2 × 123 × 72 × cos 113E
AB =
AB = 165 m (3 sf)

cos (pBAX) =

pBAX = Cos-1

= 23.7E ( 1 dp)
AD2 = 1232 + 1352 - 2 × 123 × 135 × cos 48E = 19.632 + 19.632 - 2 × 19.63 × 19.63 × cos 2.54E
AD = =
AD = 106 m (3 sf) = 0.87 m
= 3. a. Note - you could use triangle facts from several levels
with this situation. One solution is given below.
pDAX = sin-1 ( )

= 71.2E (1 dp)
ˆ pBAD = 23.7 + 71.2
= 94.9E (1 dp)
3.

Cos θ = First find pWLR using the sine rule


=
θ = cos-1

= 2.22 rad (2 dp) pWLR = sin-1


ˆ Area of segment
= Asector - Atriangle = 82.2E 1 dp or 82E (2sf)
then by co-interior angles on parallel lines
= r2θ - r2 sin θ α = 180 - 93
= 87E
= × 1.252 × 2.22 - × 1.252 × sin 2.22 and by the angle sum on a straight line
pWLR + α + θ = 180E
= 1.11 m2 (2 dp) θ = 180 - 87 - 82
b. Volume = length × cross sectional area = 11E
= 256m × 1.11 m2 ˆ bearing of S from L is 011E
= 280 m3 (2 sf) of sediment
b.
Pages 125 - 129, PRACTICAL TRIGONOMETRY PROBLEMS
- PRACTICE TEST 2
Part A
There are many possibilities, depending on the location of
Q. e.g. QR = 19.63 m, QL = 21.22 m, LR = 1.83 m
Part B
1. By the cosine rule,
pWRL = 180 - 82 - 25
cos(LQR) = = 73E
By either cosine rule or sine rule find WL
pLQR = cos-1 =

pLQR = 2.54E (2 dp)


WL =

WL = 4.14 m (from Wayne to post)


Cos Rule would have been:
WL2 = 4.292 + 1.832 - 2 × 4.29 × 1.83 × cos 73 etc

YEAR 12 MATHEMATICS
Answers
167

Pages 125 - 129 cont 2. a., b.


4. Area of the remaining (unpainted) area
= Area of Whole Circle - Area of sector
Since p’s at a point add to 2π
pAOB = 2π - 2.17
= 4.11 rad
ˆ Area = π × 4.572 - × 3.362 × 4.11

= 42.41 m2
5. Planned height of puck above C (TC) is
1.22 - 0.30 = 0.92 m

Since line PC is halfway between PL (4.9m) and PR(5.3m)

ˆ PC =

= 5.02 m
ˆ Angle of elevation pTPC =

= tan-1

= 10.4E

Pages 131 - 134, TRIGONOMETRIC EQUATIONS


Revision Summary
1. a. i. 30E × = 0.52 rad

ii. 1.99 rad


iii. 4.68 rad

b. i. 30E =

ii.

iii.
3. a. 2sin x = 1
c. i. 0.92 × = 52.7E sin x =
ii. 150.7E
iii. 300E x = sin-1

ˆ x = 30E
and x = 180-30
= 150E
ˆ x = 30E or 150E

YEAR 12 MATHEMATICS
Answers
168

Pages 131 - 134 cont


x = sin-1 (1)
b. 5cos x = 1 + 2
cos x =
x =
-1
x = cos
ˆ x = π
x = 0.927 rad, 2π - 0.927 Min value is -5
ˆ x = 0.927 rad, 5.356 rad (3dp) 5 sin( x) = -5
c. cos 2x = 0.85
First revolution
sin ( x) = -1
2x = cos-1 (0.85)
2x = 0.5548 or 2π - 0.5548
ˆ 2x = 0.5548 radians or 5.7284 radians x = sin-1 (-1)
Second revolution
2π + 0.5548, 4π - 0.5548 x =
= 6.8380 radians or 12.0116 radians
ˆ 2x = 0.5548, 5.7284, 6.830, 12.0116 ˆ x = 3π
x = 0.277, 2.864, 3.145, 6.006 radians (3 dp)
d. 2.5 sin 3x = -1.5
sin 3x =

3x = sin-1 (- 0.6)
3x = -36.87
ˆ 3x = 180+36.87, 360-36.87, 540+36.87, 720-36.87
900+36.87, 1080-36.87
x = 72.3E, 107.7E, 192.3E, 227.7E, 312.3E, 347.7E

Pages 135 - 139, TRIGONOMETRY EQUATIONS


PRACTICE TEST 1
QUESTION ONE
a. x = cos-1 (0.4)
= 66.4E
and
x = 360 - 66.4E
e. sin( x - 45) = 0.4
= 293.6E
(x - 45) = sin-1 (0.4)
ˆ x = 66.4E, 293.6E
(x - 45) = 23.6E
(radian measures = 1.159, 5.121)
Check possibilities either side of [-270, 270]
ˆ x - 45E = -360+23.6, -180-23.6, 23.6, 180-23.6
b. sin x = 3.09 - 4
x - 45E = 336.4E, -203.6E, 23.6E, 156.4E
= -0.91
x = -291.4, -158.6E, 68.6E, 201.4E
x = sin-1 (-0.91)
(Note the value -291.4 is outside the domain)
= -65.5E
Domain is 0E # x # 360E and sin is negative in the third
and fourth quadrants.
ˆ solutions are x = 245.5E, 294.5E
(radian measures = 4.285, 5.140)
c. tan x =

tan x = 1.6
x = tan-1(1.6)
x = 1.012
4. y = 5sin or y = 5sin( x) x = π + 1.012
ˆ x = 1.012, 4.154
Max value is 5 5 sin( x) = 5 (degree measures = 58.0E, 238.0E)

sin ( x) =1

YEAR 12 MATHEMATICS
Answers
169

Pages 135 - 139 cont QUESTION FIVE


QUESTION TWO If depth, d = 3m then solve
cos 2x = 0.78
2.3 cos ( ) + 3.8 =3
2x = cos-1(0.78)
2x = 38.74E
Using a graph with 2 cycles 2.3 cos ( t) = -0.8

cos ( t) =

t = cos-1 ( )

t = 1.926

t = 3.85 h
ˆ 3.85 hours (or 3h 51 min) each side of high tide has
enough water for safe passage in/out of harbour.
Graphically, this would appear on (0, 2π)

ˆ 2x = 38.74E, 360E-38.74E, 360E+38.74E, 720E-38.74E


x= 19.4E, 160.6E, 199.4E, 340.6E
(In radians, x = 0.339, 2.803, 3.480, 5.945)

QUESTION THREE
a. 33 sin(45t) = 30
sin(45t)=

45t = sin-1 ( )

45t = 65.38E
t = Pages 140 - 143, TRIGONOMETRY EQUATIONS
PRACTICE TEST 2
ˆ t = 1.453 seconds to reach 30 cm after starting at h = 0
QUESTION ONE
b. (45t)E 6 45 cycles in 360 seconds
a. θ = cos-1 (0.4)
ˆ = 8 seconds to return θ = 66.4E
and θ = 360E - 66.4
ˆ 45 eight second cycles every 360s ˆ θ = 66.4E, 293.6E
b. 2.5 sin θ = 1.5 - 1
sin θ =

θ = sin-1 (0.2)
θ = 11.5E (1 dp)
ˆ θ = 11.5E, 180E - 11.5E
ˆ θ = 11.5E, 168.5E (1 dp)
c. tan θ = 1.1 - 2.7
tan θ = - 1.6
θ = tan-1 (-1.6)
ˆ θ = -1.012
As this is outside the domain of [0, 2π]
Draw a graph of tan θ on [0, 2π] to help
= 1.453 s, 2.547 s to get to 30 cm
θ = -1.012 is matched when
θ = π - 1.012, 2π - 1.012
QUESTION FOUR
θ = 2.130, 5.271 (3 dp)
If y = 0.45
0.6 sin (512 π t) = 0.45
QUESTION TWO
sin (512 π t) = a. θ - 30E = cos-1(-0.7)
θ - 30E = 134.4E (second quadrant)
512 π t = sin-1 ( ) or 360E - 134.4E (third quadrant)
ˆ θ = 134.4E + 30E, 360E - 134.4E + 30E
ˆ θ = 164.4E, 255.6E
It is useful to check with a substitution with one of your
t = own solutions.
e.g. Evaluating cos(255.6E - 30E) = -0.69966
= 0.00053 seconds (2 sf)
= -0.7 (1 dp)

YEAR 12 MATHEMATICS
Answers
170

Pages 140 - 143 cont


b. 2θ = sin-1 (0.61)
= 0.656 rad (3 dp)
ˆ 2θ = 0.656, π - 0.656
ˆ 2θ = 0.656, 2.486
ˆ 2θ = 0.328, 1.243

QUESTION THREE

38.6 + 1.6 sin ( t) = 40E

1.6 sin ( t) = 40E - 38.6E

sin ( t) =

t = sin-1 ( )

t =

t = 2.713 days (3 dp)


ˆ during her third day.

QUESTION FOUR
First week

30 - 33 cos ( t) = 45

30 - 45 = 33 cos ( t)

= cos ( t)

cos-1 ( ) = t

ˆ t = 2.043

ˆ t = 2.043, 2π - 2.043

t = ,

t = 3.90, 8.10 (2 dp)


ˆ from the end of the 4th week, and into the beginning
of the 8th week, weekly sales of snow boards are at
least 45 boards

YEAR 12 MATHEMATICS
Normal Distribution
171

Areas Under the Normal Curve


This table gives the area under the standard normal curve between 0 and z (the shaded area).

z 0.00 0.01 0.02 0.03 0.04 0.05 0.06 0.07 0.08 0.09
0.00 0.0000 0.0040 0.0080 0.0120 0.0160 0.0199 0.0239 0.0279 0.0319 0.0359
0.10 0.0398 0.0438 0.0478 0.0517 0.0557 0.0596 0.0636 0.0675 0.0714 0.0753
0.20 0.0793 0.0832 0.0871 0.0910 0.0948 0.0987 0.1026 0.1064 0.1103 0.1141
0.30 0.1179 0.1217 0.1255 0.1293 0.1331 0.1368 0.1406 0.1443 0.1480 0.1517
0.40 0.1554 0.1591 0.1628 0.1664 0.1700 0.1736 0.1772 0.1808 0.1844 0.1879

0.50 0.1915 0.1950 0.1985 0.2019 0.2054 0.2088 0.2123 0.2157 0.2190 0.2224
0.60 0.2257 0.2291 0.2324 0.2357 0.2389 0.2422 0.2454 0.2486 0.2517 0.2549
0.70 0.2580 0.2611 0.2642 0.2673 0.2704 0.2734 0.2764 0.2794 0.2823 0.2852
0.80 0.2881 0.2910 0.2939 0.2967 0.2995 0.3023 0.3051 0.3078 0.3106 0.3133
0.90 0.3159 0.3186 0.3212 0.3238 0.3264 0.3289 0.3315 0.3340 0.3365 0.3389

1.00 0.3413 0.3438 0.3461 0.3485 0.3508 0.3531 0.3554 0.3577 0.3599 0.3621
1.10 0.3643 0.3665 0.3686 0.3708 0.3729 0.3749 0.3770 0.3790 0.3810 0.3830
1.20 0.3849 0.3869 0.3888 0.3907 0.3925 0.3944 0.3962 0.3980 0.3997 0.4015
1.30 0.4032 0.4049 0.4066 0.4082 0.4099 0.4115 0.4131 0.4147 0.4162 0.4177
1.40 0.4192 0.4207 0.4222 0.4236 0.4251 0.4265 0.4279 0.4292 0.4306 0.4319

1.50 0.4332 0.4345 0.4357 0.4370 0.4382 0.4394 0.4406 0.4418 0.4429 0.4441
1.60 0.4452 0.4463 0.4474 0.4484 0.4495 0.4505 0.4515 0.4525 0.4535 0.4545
1.70 0.4554 0.4564 0.4573 0.4582 0.4591 0.4599 0.4608 0.4616 0.4625 0.4633
1.80 0.4641 0.4649 0.4656 0.4664 0.4671 0.4678 0.4686 0.4693 0.4699 0.4706
1.90 0.4713 0.4719 0.4726 0.4732 0.4738 0.4744 0.4750 0.4756 0.4761 0.4767

2.00 0.4772 0.4778 0.4783 0.4788 0.4793 0.4798 0.4803 0.4808 0.4812 0.4817
2.10 0.4821 0.4826 0.4830 0.4834 0.4838 0.4842 0.4846 0.4850 0.4854 0.4857
2.20 0.4861 0.4864 0.4868 0.4871 0.4875 0.4878 0.4881 0.4884 0.4887 0.4890
2.30 0.4893 0.4896 0.4898 0.4901 0.4904 0.4906 0.4909 0.4911 0.4913 0.4916
2.40 0.4918 0.4920 0.4922 0.4925 0.4927 0.4929 0.4931 0.4932 0.4934 0.4936

2.50 0.4938 0.4940 0.4941 0.4943 0.4945 0.4946 0.4948 0.4949 0.4951 0.4952
2.60 0.4953 0.4955 0.4956 0.4957 0.4959 0.4960 0.4961 0.4962 0.4963 0.4964
2.70 0.4965 0.4966 0.4967 0.4968 0.4969 0.4970 0.4971 0.4972 0.4973 0.4974
2.80 0.4974 0.4975 0.4976 0.4977 0.4977 0.4978 0.4979 0.4979 0.4980 0.4981
2.90 0.4981 0.4982 0.4982 0.4983 0.4984 0.4984 0.4985 0.4985 0.4986 0.4986

3.00 0.4987 0.4987 0.4987 0.4988 0.4988 0.4989 0.4989 0.4989 0.4990 0.4990
3.10 0.4990 0.4991 0.4991 0.4991 0.4992 0.4992 0.4992 0.4992 0.4993 0.4993
3.20 0.4993 0.4993 0.4994 0.4994 0.4994 0.4994 0.4994 0.4995 0.4995 0.4995
3.30 0.4995 0.4995 0.4995 0.4996 0.4996 0.4996 0.4996 0.4996 0.4996 0.4997
3.40 0.4997 0.4997 0.4997 0.4997 0.4997 0.4997 0.4997 0.4997 0.4997 0.4998

3.50 0.4998 0.4998 0.4998 0.4998 0.4998 0.4998 0.4998 0.4998 0.4998 0.4998
3.60 0.4998 0.4998 0.4999 0.4999 0.4999 0.4999 0.4999 0.4999 0.4999 0.4999
3.70 0.4999 0.4999 0.4999 0.4999 0.4999 0.4999 0.4999 0.4999 0.4999 0.4999
3.80 0.4999 0.4999 0.4999 0.4999 0.4999 0.4999 0.4999 0.4999 0.4999 0.4999
3.90 0.5000 0.5000 0.5000 0.5000 0.5000 0.5000 0.5000 0.5000 0.5000 0.5000
4.00 0.5000 0.5000 0.5000 0.5000 0.5000 0.5000 0.5000 0.5000 0.5000 0.5000

YEAR 12 MATHEMATICS
Useful Formulae
172

FORMULAE SHEET

YEAR 12 MATHEMATICS
Pages for Extra Notes
173

YEAR 12 MATHEMATICS
Pages for Extra Notes
174

YEAR 12 MATHEMATICS
Pages for Extra Notes
175

YEAR 12 MATHEMATICS
Pages for Extra Notes
176

YEAR 12 MATHEMATICS

You might also like